Skin, Fat and Cartilage Graft 01-22 Flashcards

1
Q

A 5-year-old girl with sarcoma of the left femur undergoes resection with a limb salvage approach including an intercalary allograft for bone reconstruction. Allografts are preferred for large segmental defects of the femur and tibia compared with cancellous bone grafts due to which of the following factors?

A) Higher mechanical strength
B) Improved osteoconduction
C) Increased graft take
D) Lower cost

A

The correct response is Option A.

Large defects of the femur and tibia are typically reconstructed with intercalary allografts due to their high mechanical strength. This type of reconstruction can be augmented with an intramedullary free fibula flap, which has been shown to reduce long-term complications.

Cancellous bone grafts offer superior osteogenesis, osteoinduction, and graft take in comparison with cortical grafts and allografts. Cost is not a significant consideration in reconstructing these highly complex defects. While an allograft has no donor site morbidity compared with autologous cancellous bone grafts, this is not the primary reason for their use in the described clinical scenario.

How well did you know this?
1
Not at all
2
3
4
5
Perfectly
2
Q

A 10-year-old boy undergoes a free fibular flap reconstruction following an 8-cm resection of a chondrosarcoma on his right humerus. Which of the following best describes the primary mechanism of bone healing?

A) Endochondral ossification
B) Osteoconduction
C) Osteogenesis
D) Osteoinduction

A

The correct response is Option C.

Osteogenesis is the formation of new bone by osteoblasts at the host-graft interface. This is the primary mechanism by which a vascularized bone graft heals. The vascular pedicle permits the bone graft and its osteocytes to remain perfused independent of its recipient bed. Increased osteocyte survival leads to decreased bone remodeling and maintenance of bone mass and strength. Advantages of a vascularized bone graft include reconstruction of large bone defects (>6 to 8 cm) and the ability to withstand hostile recipient environments, such as irradiation and infection.

Osteoconduction (creeping substitution) is the process in which a material acts as a scaffold for bone growth, extracellular matrix, and blood vessels. Cortical bone grafts and demineralized bone matrix are examples of autogenous and exogenous osteoconductive methods.

Osteoinduction is the process of direct stimulation of mesenchymal stem cells at the recipient site by bone morphogenetic protein (BMP) to differentiate into osteoprogenitor cells. This is the primary mechanism of action for BMP and cancellous bone grafts.

Endochondral ossification is the process by which the cartilaginous soft callus covering a fracture is transformed into bone.

How well did you know this?
1
Not at all
2
3
4
5
Perfectly
3
Q

A 16-year-old girl requires a skin graft to the dorsal forearm after she sustained injuries in a motorcycle accident. Which of the following wound coverage techniques will lead to the LEAST secondary contraction?

A) Full-thickness skin graft, 1:4 meshed
B) Full-thickness skin graft, nonmeshed
C) Healing by secondary intention
D) Split-thickness skin graft, nonmeshed, 0.010-in thick
E) Split-thickness skin graft, nonmeshed, 0.014-in thick

A

The correct response is Option B.

Secondary contraction of a wound depends on various factors. Studies have shown that thicker grafts result in decreased scar depth and secondary contraction, whereas thinner grafts contract more. In terms of numbers, thick split-thickness skin grafts (0.03 in, 760 mm) approach similar final wound sizes as full-thickness skin grafts. Wider meshing results in more secondary contraction.

Myofibroblast differentiation is thought to be a key driver of contraction. Application of full-thickness skin grafts appears to speed up their life cycle, compared with wounds treated with split-thickness grafts. Conversely, thin grafts show a greater expression of alpha-smooth muscle actin, which persists for a longer time after grafting. Other mechanisms include decreased expression of pro-inflammatory genes one week after full-thickness grafting, such as insulin-like growth factor 1 (IGF1) and transforming growth factor beta-1 (TGFB1), when compared with wounds treated with thin split-thickness grafts.

Healing by secondary intention is associated with greater contraction than use of a graft.

How well did you know this?
1
Not at all
2
3
4
5
Perfectly
4
Q

A 20-year-old man undergoes decompressive craniectomy after a traumatic brain injury. He is now recovered from the injury. The soft tissue overlying the cranial defect is healthy and there is no communication with the paranasal sinuses. Cranioplasty with an alloplastic material is planned. A material with good osteoconductive properties is desired. Which of the following is the most appropriate choice?

A) Hydroxyapatite
B) Polyetheretherketone
C) Polymethylmethacrylate
D) Porous polyethylene
E) Titanium

A

The correct response is Option A.

The ideal material for cranioplasty is biocompatible, radiolucent, heat resistant, inexpensive, and has low risk for infection and extrusion. A healthy and well-perfused soft-tissue envelope is a prerequisite for alloplastic cranioplasty. Moreover, alloplasts in contact with sinuses have a high risk for infection.

Hydroxyapatite (HA) is a calcium phosphate compound with chemical structure similar to that of bone. Of the current alloplastic materials used for cranioplasty, it has the highest capacity for osteoconduction. It is available as a powder that is mixed with liquid to turn it into a malleable form that can be molded to the shape of the defect. Custom made HA ceramic implants can also be produced based on specifications of the defect. The major disadvantage of this material is that it is brittle and may fracture when stressed. It is sometimes used with a titanium mesh underlay to provide more structural stability.

Polymethylmethacrylate (PMMA) is a polymerized ester of acrylic acid. It is the most commonly used alloplastic material for cranioplasty. It is prepared in the operating room via an exothermic reaction; therefore, care has to be taken to protect the dura from thermal injury. Custom implants can also be produced. It can be easily contoured to the shape of the defect and has good mechanical strength. It does not have osteoconductive properties.

Porous polyethylene (PPE) is composed of high-density polyethylene microspheres with an interconnected porous structure. This allows tissue ingrowth in the implant. PPE is radiolucent and has been found to have an excellent safety profile. It does not have osteoconductive properties.

Polyetheretherketone (PEEK) is a semi-crystalline organic thermoplastic polymer. It is lightweight, strong, radiolucent, and heat resistant. Because of these properties, it is becoming a popular material for cranioplasty as a customized implant. It does not have osteoconductive properties.

Titanium is used as an alloy with aluminum and vanadium. It is biocompatible, non-corrosive, radiopaque, and heat resistant. Titanium mesh can be cut to the shape of the defect. It can also be used as a customized implant. The major advantages are its strength, ease of use, and low infection rate. The major disadvantage is its cost. It does not have osteoconductive properties

How well did you know this?
1
Not at all
2
3
4
5
Perfectly
5
Q

A 55-year-old man presents with a 5 × 5-cm open wound on the medial lower leg with healthy granulation tissue. A split-thickness skin graft from the upper thigh is planned. Which of the following dressings would result in the highest degree of pain at the donor site?

A) Alginate
B) Hydrocolloid
C) Hydrofiber
D) Petrolatum gauze
E) Semipermeable

A

The correct response is Option D.

Petrolatum-based dressings, such as Adaptic or Xeroform, are considered nonmoist dressings. Moist dressings include semipermeable (Tegaderm or OpSite), hydrofiber (Aquacel), alginate, or hydrocolloid (DuoDERM).

In a recent meta-analysis from 2018, 41 articles with 35 prospective randomized trials found that moist dressings at a skin graft donor site are associated with less pain as compared to nonmoist dressings. When examining various time points in recovery, it was found that moist dressings showed improved pain control at every time point (postoperative days 1 to 4, 4 to 7, greater than 8).

With regard to wound healing, this finding was not universal, although most studies showed that moist dressings demonstrated improvements in re-epithelialization rate and quality of healing. Infection and cosmetic outcome were also not significantly different with variable results.

How well did you know this?
1
Not at all
2
3
4
5
Perfectly
6
Q

A healthy 20-year-old woman presents with a deep abrasion injury of the left volar forearm that she sustained in a bicycle accident. Physical examination of the forearm shows exposed flexor tendons. The wound is debrided, and no vital nerves or vessels are exposed. The wound is approximately 4 × 4 cm. A bilaminate acellular dermal regenerative matrix is used to reconstruct the defect. The application of negative-pressure therapy is most likely to result in which of the following outcomes in this patient?

A) Increase in matrix take
B) Increase in timing to definitive skin graft
C) Negative-pressure therapy is not indicated
D) No change in matrix take
E) No change in timing to definitive skin graft

A

The correct response is Option A.

The application of negative-pressure therapy results in an increase in matrix take. Dermal regeneration matrix (DRM) (clinical example is Integra) is a mesh-bilayered acellular matrix composed of a cross-linked bovine tendon collagen and glycosaminoglycan dermal equivalent, and a semipermeable polysiloxane (silicone) epidermal equivalent. It is a biodegradable matrix that acts as a scaffold for fibroblast and endothelial invasion and capillary growth. DRM has been approved by the FDA (Federal Drug Administration) for use in partial- and full-thickness wounds, pressure ulcers, venous leg ulcers, surgical wounds, chronic vascular ulcers, second-degree burns, and draining wounds.

The combination of DRM with fibrin glue and postoperative negative-pressure therapy versus DRM alone in acute and chronic wounds increased the take rate from 78 to 98%. Moreover, the interval between DRM and skin transplantation was decreased from 24 to 10 days, thus decreasing the length of hospital stay (Level II evidence).

The other options are therefore incorrect. Negative-pressure therapy is commonly used in these clinical scenarios.

How well did you know this?
1
Not at all
2
3
4
5
Perfectly
7
Q

Cancellous bone grafts display which of the following characteristics compared with cortical bone grafts?

A) Improved osteogenic potential
B) Increased density with slower resorption
C) Increased structural support
D) Low concentrations of osteoblasts and osteocytes
E) Poor osteoinductive potential

A

The correct response is Option A.

Cancellous bone grafts have improved osteogenic potential when compared with cortical bone grafts. Despite significant improvements in fracture stabilization, in order to promote adequate bone union, the addition of bone grafts is occasionally necessary. These can be in the form of autografts, allografts, and synthetic bone grafts, and ortho-biologic factors. Osseous autografts can be categorized as vascularized and nonvascularized. Nonvascularized autografts can be further sub-classified as cancellous, cortical, or bone marrow aspirate. In order to choose the optimal type of graft to utilize in a specific clinical scenario, it is important to have an understanding of the properties of specific types of bone grafts as well as differences between categories of available grafts.

Bone autografts are considered the gold standard because of their immunologic compatibility as well as their osteoinductive, osteoconductive, and osteogenic healing properties. The most commonly utilized autologous bone graft is cancellous bone. When compared with cortical bone grafts, cancellous bone grafts have significant concentrations of osteoblasts and osteocytes, thus giving cancellous bone grafts superior osteogenic potential. Cortical bone grafts are dense with significant structural support, which leads to improved stability. This dense structure also leads to slower resorption compared with cancellous bone grafts, but this dense organized structure leads to lower concentrations of osteoblasts and osteocytes thus diminishing the osteogenic potential of cortical bone grafts.

How well did you know this?
1
Not at all
2
3
4
5
Perfectly
8
Q

A 9-year-old boy is undergoing cranioplasty to repair a 5 x 5-cm congenital skull defect. Which of the following is the most appropriate material to reconstruct the cranium?

A) Costal cartilage graft
B) Cranial bone graft
C) Fibula bone graft
D) Iliac bone graft
E) Rib bone graft

A

The correct response is Option B.

The most appropriate material to reconstruct the cranial defect is bone graft. Autologous bone grafts are preferred for cranioplasty in the pediatric population because the material osseointegrates and grows with the child. Alloplastic implants can become dislodged as the cranium is growing and may restrict brain growth. The preferred autologous donor site for cranioplasty is the cranium because it is in the same operative field, it is associated with the lowest morbiditiy, and the graft is least likely to resorb. Cranial bone is difficult to split in younger children because a diploic space does not begin to form until approximately 5 years of age. Cranial particulate bone graft can be harvested with a bit and brace at any age because a diploic space is not required. The partial-thickness donor sites reossify and can be used again to harvest additional graft in the future. Iliac and rib donor sites are associated with significant donor site morbidity, graft resorption, and would be less ideal for reconstruction a 5 x 5 cm defect in a 9-year-old. Costal cartilage and fibula are not used as cranioplasty materials.

How well did you know this?
1
Not at all
2
3
4
5
Perfectly
9
Q

Which of the following strategies is most effective in improving long-term viability of fat grafts?

A) Harvest fat from an abdominal donor site
B) Harvest fat with a syringe rather than a suction pump
C) Inject fat with a low-shear device
D) Process fat with telfa gauze rolling technique
E) Use tumescent solution without lidocaine

A

The correct response is Option C.

One of the biggest shortcomings of fat grafting is unpredictable graft survival. Fat grafting can be divided into three major steps: harvest, processing, and injection. Each step has the potential to influence the viability of transplanted fat. Fat donor site does not make a difference in fat graft viability. Lidocaine has been shown to inhibit the growth and metabolism of adipocytes. However, these effects disappear once the lidocaine is removed from the harvested fat and therefore does not affect long-term graft viability. Harvest technique, either with handheld syringe or a suction device, has not been shown to influence graft viability. Although some surgeons avoid centrifugation in an attempt to decrease trauma to fat cells, there is no objective evidence that centrifugation decreases fat cell viability. Trauma to fat cells, during processing or injection, affects fat graft survival. Injection of fat with low-shear devices has been shown to improve fat viability. Similarly, injection with larger cannulas has been shown in a few studies to increase fat survival. Strategies that hold promise in the future to increase fat cell viability include platelet-rich plasma, stem cell enrichment, and scaffolds.

How well did you know this?
1
Not at all
2
3
4
5
Perfectly
10
Q

A 68-year-old man is diagnosed with squamous cell carcinoma of the floor of the mouth with invasion into the tongue and mandible. A radical resection with immediate reconstruction using a vascularized free fibula bone flap is planned. A vascularized bone flap is an ideal choice in this setting due to its ability to promote bony healing through which of the following pathways?

A) Endochondral ossification
B) Osteoblastic rimming
C) Osteoconduction
D) Osteogenesis
E) Osteoinduction

A

The correct response is Option D.

Free vascularized fibula grafts are the mainstay for mandibular reconstruction in a variety of settings, from traumatic to oncologic. Like most vascularized bone grafts, they primarily heal by osteogenesis, which involves the formation of new bone by osteoblasts from both the flap and recipient site.

In contrast, non-vascularized cortical bone grafts primarily heal by osteoconduction, or creeping substitution. In this process, the bone graft acts as a template for the ingrowth of cells and blood vessels from the recipient bed and ultimately resorbs.

Cancellous bone grafts and demineralized bone matrix heal primarily by osteoinduction. In this process, bone morphogenetic protein directly stimulates mesenchymal cells at the recipient site to differentiate into osteoprogenitor cells.

Bone fractures heal, in part, by endochondral ossification. When a bone is broken, inflammation triggers chondrocytes to form a soft callus consisting of collagen and proteoglycans at the fracture site. Through endochondral ossification, disorganized woven bone replaces the soft callus to form a hard callus which is remodeled over time into highly organized cortical bone.

Osteoblastic rimming is a histologic finding indicating the presence of bony trabeculae rimmed with osteoblasts. It is seen in a variety of bone disorders.

How well did you know this?
1
Not at all
2
3
4
5
Perfectly
11
Q

A 55-year-old woman undergoes composite resection of the right mandibular body and floor of the mouth. Closure with a fibular free flap is performed. A photograph is shown. Which of the following mechanisms best describes the healing process associated with vascularized bone transfers?

A) Endochondral ossification
B) Osteochondrosis
C) Osteoconduction
D) Osteogenesis
E) Osteoinduction

A

The correct response is Option D.

Osteogenesis is the formation of new bone by cells in a flap/graft that survive the transfer. This is the primary mechanism by which a vascularized bone graft heals. The pedicle keeps the bone alive so that primary bone healing can occur between the graft and recipient site. Osteoblasts from both locations participate in the formation of new bone at the interface between graft and native bone. Advantages of a vascularized bone graft include the ability to place the graft into a hostile environment such as an irradiated wound bed and immediate structural support with shortened time to bony union compared with nonvascularized cortical grafts.

Osteoinduction refers to the direct stimulation of mesenchymal cells at the recipient site by bone morphogenetic protein to differentiate into osteoprogenitor cells. This mechanism of action is associated with the healing of cancellous bone grafts and demineralized bone matrix.

Endochondral ossification is the process by which the cartilaginous soft callus covering a fracture is transformed into bone.

Osteochondrosis refers to a family of ossification disorders in children.

Osteoconduction (creeping substitution) is the primary method by which cortical bone grafts heal. During osteoconduction, cells and blood vessels from the recipient bed grow into the graft. The bone graft becomes a template for the deposition of new bone and the graft resorbs. Neovascularization is complete by 6 to 8 weeks, but ultimate strength of cortical grafts is not seen until 6 to 12 months, at which time the graft is comparable to a vascularized bone graft.

How well did you know this?
1
Not at all
2
3
4
5
Perfectly
12
Q

A 43-year-old man undergoes wound closure with a split-thickness skin graft harvested from the left thigh. Which of the following skin appendages are the primary source of multipotent stem cells responsible for reepithelialization of the donor site?

A) Apocrine glands
B) Arrector pili
C) Eccrine glands
D) Hair follicles
E) Sebaceous glands

A

The correct response is Option D.

Hair follicles contain multipotent stem cells that are activated upon the start of a new hair cycle and upon wounding to provide cells for hair follicle and epidermal regeneration. In the hair follicle, stem cells reside in the bulge area. Bulge cells are relatively quiescent compared with other cells within the follicle but can be recruited during wound healing to support reepithelialization. Sebaceous, apocrine, and eccrine glands secrete fluids that are involved in lubricating, coating, or cooling the skin. Arrector pili are responsible for motility of cutaneous hair in response to tactile stimulation or low temperatures.

How well did you know this?
1
Not at all
2
3
4
5
Perfectly
13
Q

A 55-year-old woman is scheduled to undergo autologous fat grafting to improve the upper inner quadrant aesthetic contour of the breasts. Six months ago, she underwent bilateral mastectomy and breast reconstruction with deep inferior epigastric perforator flap coverage. The patient does not have or desire breast implants. Which of the following percentage ranges most accurately describes the likelihood of fat graft survival in this patient?

A) 10%
B) 35%
C) 60%
D) 85%

A

The correct response is Option C.

Because of demonstration of the safety, efficacy, and improvements in the harvest and preparation of fat, autologous fat grafting or liposculpting is gaining popularity to adjust aesthetic breast contour irregularities after all forms of breast reconstruction. The most common complication of fat injection remains the resorption of the grafted fat.

Fat graft volume retention has been studied previously. Employing radiologic volumetric data analysis in fat grafting for cosmetic and reconstructive breast surgery demonstrates volume retention between 50 to 80% and this retention may be time and volume dependent.

Resorption of grafted fat may be attributable to apoptosis, a reduction in adipocyte volume after transplantation and survival, or a reduction in the fluid content of the grafted mixture.

How well did you know this?
1
Not at all
2
3
4
5
Perfectly
14
Q

A 34-year-old man sustains deep second-degree burns to the forearm. He is brought to a local burn center. A skin graft is chosen as the coverage method. Which of the following management techniques offers the highest likelihood of skin graft survival in this patient?

A) Complete fascial excision
B) Early range of motion
C) Keeping the graft uncovered postoperatively
D) Placing a full-thickness graft
E) Placing a meshed graft

A

The correct response is Option E.

The principles of burn wound skin grafting dictate that first, a clean wound must be obtained. This is achieved through operative debridement and washout, to remove all necrotic eschar, and eliminate any possible source of bacteria or infection.

In most cases of second-degree burns, a complete fascial excision is not necessary, as this proceeds much deeper than the affected tissue. Instead, a tangential excision is used to remove burn eschar, in layers, until the necrotic tissue has been excised, and viable tissue remains at the base of the wound.

Following this, a split-thickness skin graft allows for greatest potential of graft survival. The use of a full-thickness graft will result in lower graft survival rates.

Meshing the skin graft prior to placement will improve survival, as it prevents accumulation of fluid or blood under the skin graft. Any collection of seroma or hematoma under the graft will prevent successful outcomes.

Placement of either a bolster dressing or a negative pressure (vacuum-assisted closure) dressing on the skin graft, will prevent mechanical shear forces, and improve graft survival rates. Thus, for the first 5 days after graft placement, any early motion or lack of dressing will result in greater risk of shear forces, and lower graft survival rates.

How well did you know this?
1
Not at all
2
3
4
5
Perfectly
15
Q

A 52-year-old woman undergoes fat grafting of the upper right breast area to correct a contour indentation after implant reconstruction. Which of the following factors will most likely increase the success of fat grafting?

A) Abdominal donor site
B) Grafting soon after harvest
C) Rinsing the fat with Ringer’s lactate
D) Ultrasonic liposuction aspiration
E) Use of centrifuged fat

A

The correct response is Option B.

There is evidence that the longer fat is exposed at room temperature, the lower the adipocyte viability. There may be complete loss of stem cell viability by 4 hours at room temperature and 24 hours at 4°C (39°F).

There is no high-level evidence suggesting that centrifuging or rinsing fat increases viability. There is also no evidence for enhanced fat survival based on donor site, such as the abdomen, thigh, or arm. Use of local anesthesia does not appear to hinder graft survival. It does appear that less mechanical trauma with low-shear harvesting instruments is helpful. Ultrasonic liposuction is designed to rupture fat cells and would likely hinder graft survival.

Fat grafting is an increasingly common tool, although results appear to be operator-dependent based on wide ranges of success in published reports. There is no absolute agreed-upon method of measuring fat survival. Most studies use volumetric analysis with imaging. More future long-term studies are needed. At this time it appears that the commonality of successful results is delicate handling of the fat.

How well did you know this?
1
Not at all
2
3
4
5
Perfectly
16
Q

A 23-year-old man comes to the office for post-traumatic cranial reconstruction 6 months after a motor vehicle collision. Physical examination shows a 5 × 4-cm full-thickness calvarial defect in the left parietal region. A titanium/hydroxyapatite cement cranioplasty reconstruction is planned. Which of the following mechanisms best describes the healing process associated with hydroxyapatite?

A) Endochondral ossification
B) Osteochondrosis
C) Osteoconduction
D) Osteogenesis
E) Osteoinduction

A

The correct response is Option C.

Restoration of craniofacial contour after infection, tumor resection, or trauma can be quite challenging. Autologous bone grafts have long been considered the gold standard because of their high likelihood of osseointegration/healing, and low risk of rejection or infection. Autologous bone grafts, however, have several drawbacks including unpredictable resorption, donor site morbidity, limited availability, prolonged operative times, and difficulty to contour. As a result, there has been an ongoing search for alternative means of reconstruction with alloplastic material.

The ideal bone substitute should be chemically inert, easily contoured, able to retain a stable shape over time, strong, resistant to infection or foreign body reaction, inexpensive, and capable of osseointegration and tissue ingrowth. Methylmethacrylate has been used frequently for calvarial reconstruction but suffers several drawbacks, including infection requiring removal of implant, plate fracture, lack of osseointegration, difficulty shaping after polymerization, and necrosis of surrounding tissue due to the exothermic nature of the curing process.

Among the most promising and well-tolerated alloplastic materials for craniofacial skeletal reconstruction are the calcium phosphate–based compounds. Hydroxyapatite [Ca(PO4)6(OH)2] forms the principal mineral component of bone and constitutes 60% of the calcified human skeleton. Calcium phosphate compounds are bioactive and capable of osteoconduction and osseointegration.

Osseointegration refers to the direct chemical bonding of an alloplast to the bony surface without an intervening fibrous tissue layer. During osteoconduction (creeping substitution), the alloplast acts as a nonviable scaffold for ingrowth of blood vessels and osteoprogenitor cells from the recipient site. Subsequently, the graft/alloplast is resorbed and replaced with new bone. This mechanism is also associated with the healing of cortical bone grafts.

Hydroxyapatite (HA) cement is a mixture of tetracalcium phosphate and dicalcium phosphate anhydrous, which react in an aqueous environment to form a paste that can be easily applied and sculpted to fit the surgical defect. HA cement sets isothermically, so there is no risk of thermal damage to the surrounding tissues. Additional benefits of HA include “off the shelf” ease of use, maintenance of volume over time, lack of radiologic scatter, and low incidence of infection.

Osteoinduction refers to the direct stimulation of mesenchymal cells at the recipient site by bone morphogenetic protein to differentiate into osteoprogenitor cells. This mechanism of action is associated with the healing of cancellous bone grafts and demineralized bone matrix. Endochondral ossification is the process by which the cartilaginous soft callus covering a fracture is transformed into bone. Osteogenesis is the process by which vascularized bone grafts heal. Viable osteocytes survive the transplantation process and produce new bone at the recipient site. Osteochondrosis refers to a family of ossification disorders in children.

How well did you know this?
1
Not at all
2
3
4
5
Perfectly
17
Q

A 46-year-old man with type 1 diabetes mellitus is evaluated for an infected foot ulcer. After adequate surgical debridement, a collagen bilayer matrix is used for coverage. Which of the following clinical factors represents the greatest risk for failure of reconstruction?

A) Anatomic location
B) Exposed bone
C) Exposed tendon
D) Polymicrobial infection
E) Type 1 diabetes mellitus

A

The correct response is Option D.

Collagen bilayer matrices have become an important option in the reconstructive ladder for lower extremity wounds. Studies have demonstrated the ability of these dermal regeneration templates to neovascularize and heal into pliable, durable coverage in an attempt to achieve stable wound healing and maintain limb length. Many of these studies were performed in the setting of diabetic wounds with exposed bone or tendon, thus each of these settings does not represent a contraindication. Adequate debridement, including clearance of any polymicrobial infection, is one of the keys to successful reconstruction.

How well did you know this?
1
Not at all
2
3
4
5
Perfectly
18
Q

Which of the following best describes the immediate mechanism of skin graft survival following placement onto the recipient wound bed?

A) Dermal contraction
B) Encapsulation
C) Inosculation
D) Revascularization
E) Serum imbibition

A

The correct response is Option E.

Serum imbibition describes the earliest stage of skin graft healing. Immediately after placement onto the wound, the graft becomes edematous and may increase its mass up to 30 to 40%. Plasma leaks from recipient bed capillaries and venules and fills the space between the wound bed and the graft. Fibrinogen within the plasma settles out and forms a fibrin bond, which helps anchor the graft to the wound. The graft passively absorbs nutrients from the underlying serum by diffusion during the first 48 hours. Metabolism within the graft becomes anaerobic and the pH level falls to 6.8. The metabolic demands of the graft also fall, with ATP levels falling 70% and glucose levels falling 80%.

Revascularization and inosculation describe the second stage of skin graft healing. These processes began shortly after graft placement, but it takes approximately 4 to 5 days for the graft to become vascularized, with maximal flow developing by day 29. Revascularization refers to direct ingrowth of new blood vessels into the graft from the underlying wound bed. Inosculation describes a process by which blood vessels from the underlying wound bed connect with existing vessels in the skin graft. More recent evidence suggests that both of these processes play a role in the development of vascularization within the skin graft: existing vasculature within the skin graft undergoes some level of degeneration. However, the acellular basal lamina persists and provides a conduit for the ingrowth of a new vascular tree from the host wound bed.

Primary contraction of a skin graft occurs immediately after harvest and is due to the recoil of elastic fibers within the dermis. Grafts with a larger amount of included dermis (e.g., full-thickness grafts) have greater primary contraction. Secondary contraction refers to contraction after the wound heals. This process is mediated by myofibroblasts and occurs more frequently in grafts with a thinner dermal component (e.g., split-thickness grafts). A larger dermal component appears to suppress proliferation of myofibroblasts within the wound. Encapsulation refers to the development of a fibrous scar capsule around a foreign device such as a breast implant.

How well did you know this?
1
Not at all
2
3
4
5
Perfectly
19
Q

A 47-year-old man undergoes split-thickness autografting for the treatment of a forearm burn. Which of the following donor site dressings is most appropriate to optimize wound healing?

A) Alginate covered with occlusive dressing for 7 days
B) Moist gauze covered with occlusive dressing for 7 days
C) Petrolatum gauze covered with occlusive dressing for 2 days, then left open to air
D) Petrolatum gauze left open to air
E) Xenograft left open to air

A

The correct response is Option A.

To optimize wound healing, a moist wound-healing environment has been shown to be superior to a dry wound-healing environment. Studies on split-thickness skin graft donor sites have not been very well designed, but many studies suggest that a moist dressing is better than a dry dressing, and several review papers support this concept. Although leaving petrolatum gauze open to air is very common and may be the most practical option in certain circumstances, it does not optimize wound healing compared with a moist dressing. The only options listed that provide a moist environment for the duration required for early reepithelialization are gauze covered with occlusive dressing and alginate dressings. Gauze covered with occlusive dressing would not work well, because conventional gauze would stick to the wound and be very difficult to remove without causing significant tissue injury. Alginate dressings are emerging as an excellent option for split-thickness skin graft donor site wounds. They are adaptable, absorptive, nonadhesive, antibacterial, and provide a moist environment for wound healing.

How well did you know this?
1
Not at all
2
3
4
5
Perfectly
20
Q

A 65-year-old man comes to the office because of a 6 × 8-cm open wound on the forearm. The wound currently has healthy granulation tissue and does not appear infected. Which of the following is an advantage of split-thickness skin grafting over full-thickness skin grafting in this patient?

A) Less metabolic demand required from wound bed
B) Less secondary contraction from elastin fibers
C) More primary contraction from myofibroblasts
D) Occurrence of inosculation before plasmatic imbibition
E) Presence of intact skin appendages within graft

A

The correct response is Option A.

Split-thickness skin grafts require less metabolic demand from the wound bed compared with full-thickness skin grafts. In situations where the wound bed may not supply adequate nutrient diffusion through the graft, a full-thickness graft may develop superficial epidermolysis.

Skin graft viability is initially based on plasmatic imbibition from the serous exudate of the wound, followed by inosculation and angiogenesis. After graft take, remodeling and scar maturation occur. Primary contracture of a graft is from elastin fibers in the dermis, whereas secondary contracture is from myofibroblast activity.

Split-thickness grafts contain epidermis and a variable amount of dermis compared with full-thickness grafts, which, by definition, contain both epidermis and the entire dermal layer. Full-thickness grafts contain intact skin appendages, whereas split-thickness grafts do not; this allows the donor site for split-thickness grafts to reepithelialize. Split-thickness grafts therefore undergo less primary and more secondary contraction.

How well did you know this?
1
Not at all
2
3
4
5
Perfectly
21
Q

A 64-year-old woman is evaluated because of the aged appearance of the dorsal surfaces of the hands. Physical examination of the hands shows lipodystrophy. Autologous fat grafting is planned to improve the cosmetic appearance. Which of the following maneuvers is most likely to decrease the risk of fat necrosis and improve fat graft viability?

A) Placement of fat grafts in small aliquots
B) Postoperative prevention of pressure on grafted regions
C) Preparation of fat grafts with centrifugation
D) Use of ultrasound-assisted liposuction for fat graft harvest

A

The correct response is Option A.

The results of fat grafting are dependent upon the surgeon’s experience and technique. Strategies to increase the take of the fat graft and decrease fat necrosis include atraumatic harvest techniques. Placement of the graft in multiple small aliquots increases the availability for vascularity, and creating a lattice-like framework when depositing the grafts avoids large-volume deposits. Placement of the grafts in a few large-volume deposits is avoided to minimize fat necrosis and creation of fatty oil cysts. Centrifugation has mixed results in fat grafting.

How well did you know this?
1
Not at all
2
3
4
5
Perfectly
22
Q

A 22-year-old woman is evaluated in the emergency department because of necrotizing fasciitis of the right upper extremity. BMI is 35 kg/m2. After adequate debridement, the entire extensor and flexor compartments are open with exposed tendon. A photograph is shown. Which of the following is the most appropriate initial method of reconstruction?

A) Anterolateral thigh free flap
B) Bilaminate neodermis
C) Full-thickness skin grafts
D) Pedicled abdominal flap
E) Split-thickness skin grafts

A

The correct response is Option B.

Although a bilaminate neodermis (Integra) artificial skin substitute has been traditionally used in the acute management of burns, there have been many successful reports incorporating its use in general reconstructive plastic surgery. Integra is a bioengineered dermal substitute consisting of a bilayer membrane system. The dermal replacement layer is composed of a cross-linked bovine tendon collagen-glycosaminoglycan (chondroitin-6-sulfate) matrix coated on one side with a synthetic polysiloxane polymer (silicone) layer. This layer functions as an epidermis to control moisture loss from the wound. The collagen matrix serves as a scaffold for ingrowth of fibroblasts, macrophages, lymphocytes, and capillaries and is ultimately replaced by host tissue. Integra “take” mirrors skin graft “take” and follows the sequence of 1) imbibition, 2) fibroblast migration, 3) neovascularization, and 4) maturation.

As the host tissue infiltrates the dermal layer, the collagen layer is biointegrated with the wound to form a vascular neodermis capable of accepting a split-thickness skin graft after a period of approximately 4 weeks. Proper patient selection and careful monitoring for infection or hematoma accumulation are crucial in this time period. After approximately 4 weeks, the silicone layer is removed and a thin (0.008- to 0.010-inch) skin graft can be applied.

Advantages of Integra use include improved cosmesis, diminished scar contracture or development of hypertrophic scar, off-the-shelf availability in large quantities, increased elasticity, and the ability to use thinner skin grafts with subsequent improvement in donor-site morbidity, scar, and time to heal. Other cited advantages include commensurate growth of the grafted tissue in children, potential avoidance of a microsurgical procedure, and placement over tendons with no significant reduction in tendon mobility. The disadvantages of Integra include its high cost, steep learning curve, need for at least two stages with subsequent increased time to final closure, potential for infection under the silicone layer, and lack of adnexal structures with patient concerns of skin dryness.

In this patient, use of Integra with delayed thin split-thickness skin grafts at 4 weeks will give the best outcome in terms of cosmesis and function. A photograph is shown. Split-thickness skin grafts are not a good option in this location because of the risk of graft loss with tendon exposure due to lack of viable paratenon and the risk of secondary contracture at the wrist and elbow. Full-thickness skin grafts would diminish the risk of secondary contracture but harvesting enough tissue to cover the entire arm would be impossible using the inguinal region as a donor site. Full-thickness grafts also require a healthier wound bed for optimal graft “take” and are not likely to heal over exposed tendons.

Free tissue transfer is an excellent technique for resurfacing the forearm, but in this patient, the thigh donor site would be difficult secondary to tissue thickness and the need for an extremely large flap. A better choice could be the scapular or parascapular donor site, perhaps in combination with a preliminary tissue expansion of that location. Obtaining enough tissue would be difficult. Banking the forearm in the abdomen would be less desirable as well, given her obesity and the need for a large surface area.

How well did you know this?
1
Not at all
2
3
4
5
Perfectly
23
Q

A 30-year-old man requires bone grafting for repair of a severe nasal deformity after a bicycle collision. Which of the following is the optimal rib donor site for ease of exposure and limited morbidity?

A) First
B) Third
C) Fourth
D) Seventh
E) Eleventh

A

The correct response is Option D.

Ribs five through seven are commonly described as the ideal sites for harvesting because of ease of access and ample bone material compared with the more cephalic or caudal ribs. The seventh rib has the added advantage of being situated over the abdominal cavity, reducing the risk for pneumothorax during harvest. Ribs five and six are also suitable, and may hide better in the inframammary crease of women.

Ribs may be harvested either whole or split depending on the amount of material needed. If a whole-rib graft is required, as long as multiple adjacent ribs are not removed, morbidity is limited. Split-rib harvest offers the advantage of eventual bone regrowth, and the area can be harvested for bone grafting again in the future if needed. The eleventh rib provides a wide expanse of medial cartilage for harvest, although it is not an ideal bone donor site.

Rib grafts offer both cortical and cancellous bone material. The cortical bone is rigid and provides immediate structural support, though final incorporation and strengthening occur over many months. A well-vascularized wound bed is required for bone healing and incorporation. A split-rib inset provides the cancellous bone surface, which is more rapidly revasculuarized. If an entire bone is needed for grafting, then a vascularized bone graft or flap with microvascular anastomosis would be better suited.

How well did you know this?
1
Not at all
2
3
4
5
Perfectly
24
Q

A 25-year-old woman is scheduled for correction of a deformity of the nasal dorsum caused by previous nasal trauma as a child. Autologous rib cartilage grafting of the dorsum is planned to correct the deformity. Which of the following complications is most likely in this patient?

A) Extrusion
B) Necrosis
C) Ossification
D) Resorption
E) Warping

A

The correct response is Option E.

The most consistent complication of cartilage grafting is the propensity to change shape or warp over time. This may be due to the presence of perichondrium or the nonuniform composition of the matrix that can affect the shape when it is placed.

Pure cartilage grafts tend to maintain shape, but grafts with an intact perichondrial layer can curl significantly and lead to unpredictable results. During septal graft harvest, care must be taken to elevate mucoperichondrial flaps in the proper plane. Likewise, auricular or costal cartilage grafts must be harvested in a subperichondrial plane. Removal of the perichondrium and softer outer cartilage layer leaves the more rigid cartilage core, which maintains shape more predictably.

Extrusion, necrosis, ossification, and resorption are not known to be affected by the presence or absence of the perichondrial layer. Fresh autologous grafts easily survive transplantation procedures and do not appear to resorb over time.

Autologous cartilage from septum, concha, or rib is considered the ideal graft material. These grafts have very low risk for infection or extrusion compared with an allograft. Cartilage grafts are tolerated well by nasal tissue.

How well did you know this?
1
Not at all
2
3
4
5
Perfectly
25
Q

A 58-year-old man is referred by an orthopedic surgeon after undergoing open reduction and internal fixation of a fracture of the ankle after falling down stairs. The repair is complicated by wound dehiscence with 2 × 2 cm of exposed tibialis anterior tendon without paratenon. A photograph is shown. After debridement of healthy tissue, reconstruction is performed using bilaminate neodermis with placement of a standard bolster dressing. At which of the following times relative to placement of the bilaminate neodermis should split-thickness skin grafting be performed?

A) At the same time
B) 1 week after
C) 2 weeks after
D) 4 weeks after
E) 6 weeks after

A

The correct response is Option D.

Integra is a bilaminate neodermal material that consists of an outer silicone sheet and an inner layer of bovine collagen and glycosaminoglycan (derived from shark cartilage), which acts as a scaffold for regeneration. Specifically, there is migration of host fibroblasts into the matrix, which subsequently proliferate and form collagen. Additionally, there is migration of endothelial cells that form a vascular network within the neodermis, which can subsequently act as a foundation to support staged split-thickness skin grafting, at which time the silicone outer layer is removed and a thin (usually 6–8/1000”) split-thickness autograft is applied. The success, or “take,” of the skin graft depends on whether the neodermis is vascularized sufficiently to maintain graft viability. A 2001 study by Moiemen et al. demonstrated clinically and histologically that this process takes 4 weeks on average, without the use of topical negative pressure.

A follow-up article from 2004 suggests this process can be accelerated from 4 weeks to an average of 7.25 days if topical negative pressure is used. Further follow-up studies based on the clinical appearance of the bilaminate neodermal matrix have suggested reduction in complication rates, improved patient tolerance, and enhanced and accelerated neovascularization with the use of topical negative pressure wound therapy. However, a study published in 2010 looking histologically at various time points could not demonstrate that topical negative pressure wound therapy accelerates neovascularization as verified by the presence of histologically patent vascular channels. The authors found that the median percentage of the template depth that demonstrated canalized channels was 0%, 20%, 61%, and 80% for days 7, 14, 21, and 28, respectively.

Because this question pertains to the use of bilaminate neodermal matrix with the use of a standard bolster dressing and not negative pressure wound therapy, most existing evidence indicates that the most appropriate timing is 4 weeks after placement of the bilaminate neodermis.

How well did you know this?
1
Not at all
2
3
4
5
Perfectly
26
Q

A 30-year-old man who works as a laborer comes to the emergency department after sustaining avulsion injuries over the dorsum of the index, long, and ring fingers of his right dominant hand after being caught in a press. The wounds measure 3 × 1 cm each. Physical examination shows exposed extensor tendons from the proximal interphalangeal joint to the dorsal fold. Artificial dermis was placed. Which of the following is the most appropriate method of reconstruction?

A) Full-thickness skin graft
B) Radial artery free flap
C) Reverse radial forearm flap
D) Split-thickness skin graft
E) No further treatment

A

The correct response is Option D.

Split-thickness and full-thickness skin grafting directly over extensor tendons will lead to scarring due to lack of paratenon. Paratenon is necessary for tendon gliding. The use of artificial dermis can allow for tendon gliding. Artificial dermis has been shown to result in favorable aesthetic and functional outcomes. Although a second surgery is required, it results with lower donor site morbidity. Once the dermal template is adherent, it can then be grafted. The correct response is split grafting because the artificial dermis acts as the deep layer. Split-thickness grafts alone would not be useful where padding is required. Split grafts have a higher contracture rate than full grafts. Also, split grafts alone would not be beneficial because of the high potential for adherence. A full-thickness graft with its dermal elements would not be necessary in this case because the artificial skin provides the dermal elements. Full-thickness graft does not contract as much as split-thickness grafts. In this situation, the split graft contracture would result in a smaller, potentially more aesthetic scar.

Re-epithelialization is usually reserved for wounds measuring 1 cm or less; therefore, it would be inappropriate to not give additional treatment. Although the wound would close, the result would be less aesthetic with tendon adherence and potential dysesthesia, and it would require a longer period to heal. The reversed radial forearm flap would not be necessary and would not be long enough to reach the fingertips as well. Similarly, a radial forearm free flap would not be necessary.

How well did you know this?
1
Not at all
2
3
4
5
Perfectly
27
Q

A 30-year-old man comes to the emergency department after falling on his outstretched left hand 6 hours ago. He injured the same hand 3 years ago and underwent open reduction and internal fixation and distal radius bone grafting to repair a metacarpal fracture of the index finger. Physical examination and x-ray study obtained today show an exposed, fractured plate. He undergoes hardware removal and multiple washouts, resulting in a 1-cm bone defect. Reconstruction of the defect is planned using bone graft from Gerdy’s tubercle of the tibia. The cortical window should be made just proximal and medial to which of the following muscles?

A) Extensor digitorum longus
B) Extensor hallucis longus
C) Peroneus longus
D) Popliteus
E) Tibialis anterior

A

The correct response is Option E.

The lateral tibial tubercle (Gerdy’s tubercle), on the proximal lateral tibia, is an excellent source of cancellous bone graft and would provide ample graft for use in this patient. In the patient described, the ipsilateral distal radius was harvested in the original surgery and is not available. Patient factors including obesity may make other bone graft donor sites such as the iliac crest less appealing.

Gerdy’s tubercle is located between the tibial tubercle and patellar tendon insertion to the medial side and the head of the fibula to the lateral side. Bone can be harvested through a 3- to 4-cm oblique incision, and the cortical window is made just proximal and medial to the tibialis anterior origin.

The popliteus muscle is posterior to the knee. The extensor hallucis longus and extensor digitorum longus muscles originate more distally on the tibia. The peroneus longus originates from the fibula, not the tibia.

How well did you know this?
1
Not at all
2
3
4
5
Perfectly
28
Q

A 48-year-old man with HIV infection comes to the office requesting facial rejuvenation because of the accelerated changes he has seen since his HIV-positive status was identified. He is receiving antiretroviral therapy, has a nondetectable viral load, and normal T-cell count. Physical examination shows significant facial lipoatrophy despite the patient appearing somewhat overweight with a BMI of 27 kg/m2. The patient desires fat transfer to correct this deformity. Which of the following best describes this patient’s risk evaluation when compared with HIV-negative patients?

A) Decreased satisfaction rate
B) Higher risk of infection
C) Increased bruising
D) Increased fat graft resorption
E) Similar risk profile

A

The correct response is Option E.

HIV-positive patients with nondetectable viral loads and normal T-cell counts are at no higher risk for infection with transdermal procedures than HIV-negative patients. While finding adequate subdermal fat for fat harvesting can be challenging in some HIV patients because of “wasting,” or loss of fat, in patients with a normal or high BMI, fat for transfer is generally not an issue. No studies have shown higher rates of transferred fat reabsorption in HIV, while studies have shown long-term (12-month) retention using CT scans as an objective measure. Wasting is a hallmark of late-stage AIDS. Antiretroviral therapy is a mainstay of treatment for HIV-infected patients, generally improves wasting, and has dramatically improved the health and longevity of HIV-positive patients. Nonetheless, antiretrovirals are a major contributing factor to facial lipoatrophy even in stable HIV-positive patients with low or nondetectable viral loads and normal and near normal CD4 cell levels. While facial fat grafting in any patient is somewhat unpredictable, for healthy HIV-positive patients with adequate body fat, it is a reasonable approach to facial rejuvenation in patients with lipoatrophy.

Patient satisfaction and bruising should show the same variability as the HIV-negative patient.

How well did you know this?
1
Not at all
2
3
4
5
Perfectly
29
Q

A 45-year-old man undergoes excision of a basal cell carcinoma of the mid cheek. A 3.6 × 4.7-cm oval skin defect with exposed subcutaneous fat is present. Skin grafting is planned. Which of the following is most likely to minimize long-term scar contracture?

A) Choosing a donor site with appropriate color match
B) Defatting of the skin graft
C) Harvesting with a mechanical dermatome
D) Immobilizing with negative pressure wound therapy
E) Including the entire dermis

A

The correct response is Option E.

The defect is a full-thickness skin defect with exposed subcutaneous fat. Coverage with a local flap and skin graft are possible reconstructive options. If skin grafting were to be performed, then a full-thickness skin graft would minimize the amount of long-term contracture (secondary contracture). A skin graft can be either split- or full-thickness. Split-thickness grafts can be harvested with a mechanical dermatome, or free hand with a scalpel. Split-thickness skin grafts do not include the entire thickness of the dermis and have less initial contracture at the time of harvest (primary contracture). Full-thickness skin grafts contain both the epidermis and the dermis and would have less long-term contracture.

Harvesting with a mechanical dermatome is useful for obtaining split-thickness skin grafts. Typical thicknesses may range from 8/1000th of an inch to 14/1000th of an inch.

Adequate immobilization of a skin graft is critical for “take” of the graft. Negative pressure wound therapy can be employed to promote adherence of the graft to the wound bed.

Defatting of the skin graft in the case of a full-thickness graft is important to optimize graft “take” in the early postoperative period.

Obtaining skin from a donor site with a close color match to the recipient site is an important aesthetic consideration, but does not affect long-term contracture.

How well did you know this?
1
Not at all
2
3
4
5
Perfectly
30
Q

A 29-year-old woman is evaluated because of a bump that developed on the nasal dorsum 6 months after she underwent reconstruction with an onlay auricular cartilage graft to treat a post-traumatic nasal deformity. Which of the following is the most likely cause of this patient’s complication?

A) Infection
B) Ossification
C) Rejection
D) Resorption
E) Warping

A

The correct response is Option E.

Warping is one drawback of auricular cartilage grafts. They can curl over time. Warping is more common when the perichondrial layer is left on the graft. Pure cartilage grafts tend to warp less often. Auricular cartilage and pure cartilage warping would occur gradually over a period of weeks or months.

Cartilage is a versatile graft material that is useful for providing structural support or treating contour problems. The rib, nasal septum, and ear are the most common donor sites. Grafts are primarily composed of hyaline and elastic cartilage, with only a 1 to 10% volume of cells, so they can easily survive transplantation. Graft nutrition relies on diffusion of nutrients through the matrix of proteoglycans, interstitial fluid, and chondrocytes.

Early contour deformities could result from malposition or shifting of the graft. In this clinical case, the deformity occurs at 6 months postoperatively, when the graft should be stable in the soft-tissue envelope. Rejection is not likely with autologous cartilage and would likely occur with extrusion or fluid drainage. Rejection or failure to incorporate would typically be associated with an inadequately vascularized soft-tissue envelope or infection, and these complications would be expected within the first few weeks.

Resorption can occur gradually but is uncommon. A contour indentation would be more likely to result than a protruding bump. Ossification of the graft is not a known complication.

How well did you know this?
1
Not at all
2
3
4
5
Perfectly
31
Q

Which of the following bone substitutes has the capacity for osteoconduction and osseointegration?

A) Hydroxyapatite
B) Polymethylmethacrylate
C) Porous polyethylene
D) Silicone
E) Titanium

A

The correct response is Option A.

Osteoconduction is the ability of a material to encourage bone to grow toward and along its surface. Osseointegration is defined as the direct chemical bonding of an alloplast to the surface of bone without an intervening layer of fibrous tissue. These qualities are important in identifying an appropriate bone substitute in craniofacial reconstruction.

Hydroxyapatite is a bone substitute that has capacity for both osteoconduction and osseointegration and is the base for many of most widely used bone substitutes. Hydroxyapatite is the principal mineral component of bone and comprises 60% of the calcified human skeleton. It has been used clinically for more than 25 years. It is biocompatible, and all forms are resistant to absorption after implantation.

Silicone products do not osseointegrate or osteoconduct.

Polymethylmethacrylate causes an extreme exothermic reaction associated with the setting process and is deleterious to adjacent bone and soft tissue, even with vigorous saline irrigation. It can be designed or shaped ex vivo to avoid thermal injury.

Titanium osseointegrates, but it does not osteoconduct. It provides ideal protection and reconstruction in certain clinical situations where infection is of higher concern.

High-density porous polyethylene implants have pore sizes ranging from 100 to 300 ?m. These aid in tissue ingrowth and implant fixation. These implants may be coated with polyhydroxyethylmethacrylate and calcium hydroxide. The hydrophilic nature and calcium coating result in osteoconductivity and a fibro-osseous matrix.

How well did you know this?
1
Not at all
2
3
4
5
Perfectly
32
Q

A 40-year-old man desires correction of the appearance of his nose after traumatic injury 14 months ago. Examination shows collapse of the nasal bones and mid vault. The patient is concerned about additional scarring and donor site pain and requests a procedure with the least amount of donor site morbidity. Which of the following options is most appropriate for this patient?

A) Bone allograft
B) Costal cartilage graft
C) Iliac crest graft
D) Split calvarial graft
E) Temporal fascia graft

A

The correct response is Option A.

Freeze-dried bone allograft has been used extensively for orthopedic trauma and tumor reconstruction and has been demonstrated to be safe for nasal augmentation. The advantage of allograft is the avoidance of donor site harvesting and morbidity. Fresh autografts probably have more osteoinductive capacity and are likely to incorporate donor bone beds more thoroughly. Although this is important in bone grafting to injured bone, such as in a tibia fracture, it is less important in nasal grafting to a nasal soft-tissue bed. Freeze-dried allografts, much like acellular dermal grafts, are extensively processed to denture all cellular elements and therefore do not elicit immunologic rejection response. Irradiated costal cartilage allografts have also been used with success for nasal reconstruction. There is also a rare chance of disease transmission from the cadaveric donor.

Autologous costal cartilage is one of the more commonly used graft materials for nasal reconstruction, although both donor site scarring and pain are prominent. Iliac crest is a useful graft site for cortical and cancellous bone, though the shape is not ideal for nasal contouring. Donor site pain is an issue as well. The same limitations apply to split calvarial grafts, which are most useful when a bicoronal incision has already been used for craniofacial exposure.

Temporal fascia is a versatile graft source, especially when wrapped around diced cartilage. The resulting graft is pliable, soft, and has been reported to have minimal absorption. Although it is an excellent choice for this case, it does require a scalp donor site, which this patient does not want. Alloplastic materials such as silicone are also used, though they are prone to extrusion over time.

How well did you know this?
1
Not at all
2
3
4
5
Perfectly
33
Q

An 18-year-old woman with a history of cleft lip and palate presents for secondary alveolar bone grafting. An iliac crest bone graft is planned. Which of the following characteristics of iliac crest bone graft is an advantage over the use of bone morphogenetic protein in this patient?

A) Greater volume of graft material
B) Osteoconductive properties
C) Osteoinductive properties
D) Reduced operative time
E) Reduced recovery time

A

The correct response is Option B.

The majority of cleft lip and cleft palate patients undergo secondary bone grafting of the alveolar cleft between the ages of 8 and 12 years. A commonly used source of graft material, iliac crest bone, is associated with morbidity including significant pain, impaired ambulation, and prolonged recovery. Some authors have proposed the use of bone morphogenetic protein for alveolar cleft closure. Advantages to this technique include reduced operative time, quicker recovery, and a greater volume of graft material, which can be limited when harvesting iliac crest bone graft in smaller children. Osteogenesis requires both osteoconductive materials and osteoinductive factors. Iliac crest bone graft displays necessary properties, while bone morphogenetic protein provides significant osteoinductive properties, but requires an additional carrier, such as demineralized bone putty, for osteoconduction. Bone morphogenetic protein is not FDA-approved for patients younger than 12 years of age.

How well did you know this?
1
Not at all
2
3
4
5
Perfectly
34
Q

A 33-year-old woman sustains trauma to the right thigh. She undergoes debridement of the wounds. Two days later, the right anterior thigh has a 15 × 25-cm wound with areas of exposed fat and muscle. Which of the following is the most appropriate intervention to achieve wound closure?

A) Free latissimus dorsi flap
B) Full-thickness skin graft
C) Local fasciocutaneous flap
D) Negative pressure wound therapy
E) Split-thickness skin graft

A

The correct response is Option E.

Split-thickness skin grafts can provide wound coverage over a large area. A mechanical dermatome is often used for obtaining split-thickness skin grafts. Typical thicknesses may range from 8/1000th of an inch to 14/1000th of an inch. The graft can be meshed in various ratios such as 1:1.5, 1:2, and 1:3 to allow for a larger area of coverage per unit of harvested skin. It is important that the underlying wound bed be viable and free of necrotic tissue or infection in order to allow for healing of the skin graft (“skin graft take”). Adequate immobilization of a skin graft is important for take of the graft, and can be achieved with negative pressure wound therapy, or tie-over-bolster dressing. The thigh has an abundant amount of soft tissue and muscle, which is why skin grafts are often sufficient for wound coverage rather than flaps.

The patient has a complex wound of the anterior thigh that is best described as a degloving injury in which the skin has been sheared off of the underlying tissues. Undermining of the skin is a hallmark of this type of injury. This type of injury disrupts the blood supply to the skin and can result in tissue ischemia and necrosis. In the acute period, it can be difficult to determine the extent of tissue injury as the skin viability evolves over this time such that areas of marginal blood supply may worsen and progress to full-thickness necrosis. Before definitive wound closure can be achieved, it is critical to debride all devitalized tissue such that there is a healthy viable wound bed. Hence, performing repeat debridement is often necessary. In some cases, debriding the surrounding skin as well as the underlying fat and muscle is required to remove all necrotic tissue. Debridement should continue until healthy tissue is encountered, which can be identified by visual inspection and the presence of punctate bleeding.

The surgeon must consider several things when deciding between a flap and a graft. The reconstructive ladder may be used as a guide for management in this case. The defect is too large to achieve primary closure. The use of negative pressure wound therapy for such a large wound may be helpful as a temporary measure, but as a method of definitive wound closure would result in healing by secondary intention, scarring, and prolonged wound care. A full-thickness skin graft is not appropriate because of the large size of the defect and the amount of skin graft that would be required. A full-thickness graft would result in a major defect in another part of the body that would require primary closure. A local fasciocutaneous flap for such a large defect would require significant mobilization of tissue, and similarly, would result in a large donor site defect that would require grafting. A free flap is not necessary when there is viable soft tissue in the wound base. There is no exposed bone, tendon, nerves, blood vessels, or significant dead space, which would make a stronger argument for a flap-over-skin graft. Although not provided as an option in this question, the use of biosynthetic materials or dermal matrix tissues has been reported in the literature as an intermediate step to skin grafting, but it is important to consider the necessity of these materials in effecting outcomes in light of the significant cost of using them.

How well did you know this?
1
Not at all
2
3
4
5
Perfectly
35
Q

A 67-year-old woman comes to the office for evaluation of abscesses on her face 5 weeks after undergoing autologous fat grafting for augmentation of the cheek. Physical examination shows multiple erythematous nodules, areas of induration, and microabscesses on both cheeks. Temperature is 99.5°F (37.5°C). Liposuction donor sites show no abnormalities. A 1-week course of ciprofloxacin prescribed by the patient’s family doctor failed to resolve the problem. Results of Gram stain and routine culture and sensitivity are negative. Which of the following is the most likely diagnosis?

A) Atypical mycobacterial infection
B) Herpes zoster infection
C) MRSA infection
D) Mucocutaneous candidiasis
E) Staphylococcus epidermidis infection

A

The correct response is Option A.

Mycobacteria are ubiquitous in soil and water, and infections caused by these organisms can complicate aesthetic liposuction and autologous fat-grafting procedures. The postoperative symptoms include cellulitis, abscess formation, draining sinuses, and postoperative wound infection. Patients often do not have fever, chills, or other signs of systemic infection. While patients on immunosuppressive medications seem to be at higher risk, the problem also occurs in patients with healthy immune systems. More than 50% of patients will test negative for acid-fast bacilli, in addition to negative routine culture results. Polymerase chain reaction assay testing is now available for the most common species of nontuberculous mycobacterium, and is a useful rapid screening test for patients suspected of having this diagnosis.

Bacterial infections (MRSA) would tend to show symptoms earlier, have systemic signs, and are usually easily cultured. Herpetic infections usually present with fluid-filled vesicles earlier in the postoperative course, and often have pain as a primary complaint. There is no history of previous injectable fillers, or implants, to suggest a biofilm-related infection in this patient. Mucocutaneous candidiasis is a chronic disease of the skin, nails, and mucosal surfaces.

How well did you know this?
1
Not at all
2
3
4
5
Perfectly
36
Q

A healthy 30-year-old woman undergoes autologous fat grafting of the buttocks. The patient receives a dose of antibiotics before incision with sterile skin preparation and draping. Tumescent liposuction is performed. Gravity separation of the fat is performed intraoperatively, and 350 mL of fat is reinjected into each buttock. Postoperatively, the patient reports some areas of discrete tenderness, firmness, and limited erythema. Incision and drainage shows oily, cloudy fluid. Which of the following is the most likely cause of this patient’s complication?

A) Inadequate administration of antibiotics preoperatively
B) Inappropriate length of time for tumescent effect before aspiration
C) Lack of antibiotic administration postoperatively
D) Method of fat injection delivery
E) Use of gravity fat separation rather than centrifugation

A

The correct response is Option D.

The most likely cause of this patient’s complication is inadequate attention to injection delivery of microaliquots of fat, leading to fat necrosis. Liposculpting, or liposuction and fat grafting, for buttock contour improvement is increasing in popularity and becoming a frequently performed procedure. Good results can be obtained, and patient satisfaction can be high. However, complications may also occur and should not be ignored. Proper technique is an essential component of effective liposculpting. Delivery of overly large amounts of fat into inadequate substrate can lead to inadequate revascularization and fat necrosis, described in the scenario as “tenderness, firmness, and drainage of cloudy, oily fluid.” Superinfection of nonviable tissue can occur, creating “limited erythema” responsive to “a short course of oral antibiotics,” but the most likely cause is not related to preoperative or postoperative antibiotics because the patient received what can be considered appropriate antibiotic therapy for a “clean,” elective case. Neither the described 3:1 fat-to-fluid ratio nor the use of gravity fat separation is considered an inappropriate liposuction technique.

How well did you know this?
1
Not at all
2
3
4
5
Perfectly
37
Q

A 32-year-old woman comes to the office because of capsular contracture of the right breast. She underwent bilateral augmentation mammaplasty with saline implants 9 years ago. Revision surgery using simultaneous implant exchange with fat grafting is planned. Which of the following is the most likely fat retention volume over a 6-month period in this patient?

A) 25%
B) 45%
C) 65%
D) 85%

A

The correct response is Option C.

Previously published work using radiologic volumetric data analysis with fat grafting for cosmetic augmentation mammaplasty demonstrates volume retention over 6 months of 64 ± 11%. Such loss of breast volume may be attributable to an element of apoptosis, a reduction in adipocyte volume after transplantation and survival, or a reduction in the fluid content of the grafted slurry. In reality, all three of these factors are likely to contribute to volume reduction over time.

38
Q

A 56-year-old man with type 1 diabetes mellitus comes to the office because of a foot ulcer over the first metatarsal head without evidence of exposed bone or osteomyelitis. A bioengineered cellular bilayered skin substitute (Apligraf®) is considered for treatment. Which of the following is the primary mechanism of action of this skin substitute to stimulate wound healing in this patient’s ulcer?

A) Creation of a moist environment through creation of a barrier
B) Enzymatic degradation of the wound bed
C) Inhibition of growth factor release
D) Integration of the graft and creation of neo-dermis
E) Release of matrix proteins

A

The correct response is Option E.

Several biologic dressings have been FDA-approved for the treatment of diabetic foot ulcers. Apligraf® (Organogenesis, Inc., Canton, MA) and OrCel® (Ortec International, New York) are bilayered constructs of bovine collagen with human keratinocytes and fibroblasts. Although it appears like normal skin, it does not take like a skin graft, but rather the viable cells in the construct release and stimulate growth factors and matrix proteins to encourage wound healing.

Neo-dermis creation is the primary function of Integra ™ (Integra LifeSciences Corp, Plainsboro NJ) which is an acellular construct composed of collagen and chondroitin-6-sulfate covered with a silicone top layer to prevent evaporative losses. It does not have any active cells or release growth factors or matrix proteins. Enzymatic degradation of a wound bed is not a function of biologic grafts, but is the main function of topical wound treatments like the collagenase Santyl® (Healthpoint Biotherapeutics, Fort Worth TX).

39
Q

A 35-year-old man undergoes reconstruction of a degloving injury of the palmar surface of the hand. Full-thickness skin grafting from the groin is planned. Which of the following structures of healthy palmar skin will be missing from this graft?

A) Dermal neural mechanoreceptors
B) Dermal sweat ducts and glands
C) Irregular dermal-epidermal border
D) Pilosebaceous structures
E) Sensory and autonomic nerve fibers

A

The correct response is Option A.

The skin on the palm has specialized, encapsulated nerve endings called Meissner corpuscles in the dermal papillae and Vater-Pacini corpuscles in the deep dermis. These special dermal neural mechanoreceptors are unique to glabrous skin. Skin grafts from nonglabrous donor sites lack this feature and will have poor return of sensibility.

There are a few features that are common between palmar and regular skin. Both contain intraepidermal nerve endings and sweat ducts and glands. Both have an irregular border between the basal layer of the epidermis and dermis, at which juncture are the dermal papillae and epidermal rete ridges. A network of blood vessels and sensory and autonomic nerve fibers in the dermis is shared by all skin.

Palmar skin has deeper papillae and ridges, as the keratin layer is considerably thicker; however, glabrous donor sites for grafting are limited. Finally, pilosebaceous structures are absent in the palm.

40
Q

A 77-year-old woman undergoes excision of a basal cell carcinoma of the mid cheek. Physical examination shows a 4 × 4-cm circular skin defect with exposed subcutaneous fat. Reconstruction with skin grafting is planned. Use of which of the following is most appropriate to minimize long-term graft contracture?

A) Cultured epidermal autografting
B) Full-thickness skin grafting
C) Split-thickness skin grafting with meshing
D) Split-thickness skin grafting with no meshing

A

The correct response is Option B.

The defect is a full-thickness skin defect with exposed subcutaneous fat. A local flap or skin grafting are possible reconstructive options. If skin grafting were to be performed, then a full-thickness skin graft would minimize the amount of long-term contracture (secondary contracture). A skin graft can be either split-thickness or full-thickness.

Full-thickness skin grafts contain both the epidermis and the dermis and would have less long-term contracture. Defatting of the skin graft is important in the case of a full-thickness graft to optimize graft “take” in the early postoperative period. Various donor sites are available for skin grafting.

Cultured epidermal autografts (CEAs) are useful when there are limited areas on the body to be used as skin graft donor sites, such as in a massive burn patient. However, CEAs are costly, and the resulting skin is often very thin and fragile. Therefore, judicious use of CEAs is warranted.

Split-thickness grafts can be harvested with a mechanical dermatome or a free hand with a scalpel. Split-thickness skin grafts do not include the entire thickness of the dermis and have less initial contracture at the time of harvest (primary contracture) but are expected to have long-term contracture (secondary contracture). Harvesting with a mechanical dermatome is useful for obtaining split-thickness skin grafts. Typical thicknesses may range from 8/1000 to 14/1000 of an inch. Meshing allows for a broader surface area to be covered by a skin graft but will lead to greater contracture within the open interstices of the graft.

Of note for deeper wounds extending below the superficial musculoaponeurotic system, evaluation of facial motor nerve function and status of the parotid duct may be important considerations.

41
Q

A 35-year-old Asian woman desires dorsal augmentation to achieve a more “Western” nose. Autologous rib cartilage grafting is planned. Which of the following complications is most common with this procedure?

A) Extrusion
B) Infection
C) Pneumothorax
D) Resorption
E) Warping

A

The correct response is Option E.

The most consistent complication of costal cartilage grafting is the propensity to warp or change shape over time. This may be due to the presence of perichondrium or the nonuniform composition of the matrix that can affect the shape when it is placed.

Although there is a risk of pleural violation and pneumothorax, these complications are uncommon and can be recognized intraoperatively with the Valsalva maneuver. Through the same exposure for harvesting the rib graft, a red rubber catheter is placed in the chest to evacuate intrathoracic air. As long as the visceral pleura remain intact, a thoracostomy tube is unnecessary.

Fresh autologous grafts easily survive transplantation procedures and do not appear to resorb over time.

Autologous cartilage from septum, concha, or rib is considered the ideal graft material. These grafts have very low risk of infection or extrusion compared with an alloplast. Cartilage grafts are tolerated well by nasal tissue.

42
Q

A 24-year-old Chinese-American, right-hand-dominant man undergoes cadaveric hand transplantation after traumatic amputation at the right wrist from a machinery accident. The donor is a Hispanic man. Which of the following terms best describes the antigenicity of the transplant?

A) Allograft
B) Autograft
C) Isograft
D) Xenograft

A

The correct response is Option A.

Any tissue transplantation from another genetically nonidentical human is termed allograft, previously referred to as homograft. As these transplanted tissues are immunologically different from the recipient, they will eventually undergo rejection from the host immune system without immunosuppressive medications. There are many distinct antigens responsible for the rejection process, the most important of which are the major histocompatability complex (MHC) antigens, known as HLA-1 and HLA-2, which reside on the surface of cells.

Autograft implies that the donor tissues come from the same patient – such as the common autologous skin graft.

Isograft comes from a genetically identical donor, namely an identical twin. While such donations are very rare, it is of important historical note that the first human kidney transplantation was performed by Dr. Joseph Murray between identical twin donor and recipient. In this clinical scenario, as one patient is Asian and the other Hispanic, it is clear that the transplant is not an isograft.

Xenograft is a cross-species graft. Porcine skin grafts are commonly used as temporary skin substitutes to promote granulation formation in difficult wound beds.

43
Q

An otherwise healthy 7-year-old boy is brought to the office because of a 4-year history of progressive soft-tissue atrophy of the left forehead with coup de sabre. Examination shows skin atrophy, discoloration, and alopecia at the hairline in the V1 distribution. Which of the following is the most appropriate method to correct the deformity?

A) Forehead contouring with a calcium phosphate ceramic
B) Free tissue transfer
C) Implantation of a custom silicone prosthesis
D) Injection of hyaluronic dermal filler
E) Serial fat grafting

A

The correct response is Option E.

Parry-Romberg syndrome can be associated with underlying neurologic disease. The symptoms generally first manifest in the first or second decade of life and can involve bone, cartilage, fat, or skin. Free tissue transfer performed once the disease has stabilized has been shown to be a highly satisfactory correction. However, in recent years, fat grafting has become a more utilized option. Although more procedures are required, it can begin in younger patients even if the disease is still active. It also has demonstrated high patient satisfaction.

Hyaluronic acid dermal fillers are not FDA-approved in children and are unlikely to provide long-term correction. Custom prostheses or calcium phosphate ceramics would not reconstruct with like tissue; one would be less likely to have a stable, soft aesthetically pleasing result.

44
Q

A 25-year-old man is scheduled to undergo reconstruction of a 5 × 5-cm, full-thickness calvarial defect. A titanium/hydroxyapatite cement cranioplasty reconstruction is planned. Which of the following mechanisms best describes the healing process involved?

A) Endochondral ossification
B) Osteochondrosis
C) Osteoconduction
D) Osteogenesis
E) Osteoinduction

A

The correct response is Option C.

Restoration of craniofacial contour after infection, tumor resection, or trauma can be quite challenging. Autologous bone grafts have long been considered the gold standard due to their high likelihood of osseointegration/healing and low risk of rejection or infection. Autologous bone grafts, however, have several drawbacks, including unpredictable resorption, donor site morbidity, limited availability, prolonged operative times, and difficulty in contouring. As a result, there has been an ongoing search for alternative means of reconstruction with alloplastic material.

The ideal bone substitute should be chemically inert, easily contoured, retain a stable shape over time, strong, resistant to infection or foreign body reaction, inexpensive, and capable of osseointegration and tissue ingrowth. Methylmethacrylate has been used frequently for calvarial reconstruction but suffers several drawbacks, including infection requiring removal of implant, plate fracture, lack of osseointegration, difficulty shaping after polymerization, and necrosis of surrounding tissue due to the exothermic nature of the curing process.

Some of the most promising and well-tolerated alloplastic materials for craniofacial skeletal reconstruction are the calcium phosphate-based compounds. Hydroxyapatite [Ca(PO4)6(OH)2] forms the principal mineral component of bone and constitutes 60% of the calcified human skeleton. Calcium phosphate compounds are bioactive and capable of osteoconduction and osseointegration.

Osseointegration refers to the direct chemical bonding of an alloplast to the bony surface without an intervening fibrous tissue layer. During osteoconduction (creeping substitution), the alloplast acts as a nonviable scaffold for ingrowth of blood vessels and osteoprogenitor cells from the recipient site. Subsequently, the graft/alloplast is resorbed and replaced with new bone. This mechanism is also associated with the healing of cortical bone grafts.

Hydroxyapatite cement is a mixture of tetracalcium phosphate and dicalcium phosphate anhydrous which reacts in an aqueous environment to form a paste that can be easily applied and sculpted to fit the surgical defect. Hydroxyapatite cement sets isothermically, so there is no risk of thermal damage to the surrounding tissues. Additional benefits of hydroxyapatite include “off the shelf” ease of use, maintenance of volume over time, lack of radiologic scatter, and low incidence of infection.

Osteoinduction refers to the direct stimulation of mesenchymal cells at the recipient site by bone morphogenetic protein to differentiate into osteoprogenitor cells. This mechanism of action is associated with the healing of cancellous bone grafts and demineralized bone matrix.

Endochondral ossification is the process by which the cartilaginous soft callus covering a fracture is transformed into bone. Osteogenesis is the process by which vascularized bone grafts heal. Viable osteocytes survive the transplantation process and produce new bone at the recipient site. Osteochondrosis refers to a family of ossification disorders in children.

45
Q

Which of the following techniques minimizes fat necrosis and increases the viability of fat grafting?

A) Avoidance of pressure on grafted areas postoperatively
B) Avoiding placement of the grafts in irradiated tissue
C) Centrifuging the fat grafts
D) Placing the graft in multiple small volumes in a lattice-like framework
E) Using ultrasound-assisted liposuction to harvest grafts

A

The correct response is Option D.

The results of fat grafting are dependent upon the surgeon’s experience and technique. Strategies to increase the take of the fat graft and decrease fat necrosis include atraumatic harvest techniques. Placement of the graft in multiple small aliquots increases the availability for vascularity and creating a lattice-like framework when depositing the grafts avoids large-volume deposits. Placement of the grafts in a few large-volume deposits is avoided to minimize fat necrosis and creation of fatty oil cysts. Fat grafting has been refuted for soft tissues that have been affected by radiation.

46
Q

Which of the following is the most common long-term complication of auricular cartilage harvesting?

A) Cauliflower ear
B) Hematoma
C) Necrosis
D) Perichondritis
E) Sensory impairment

A

The correct response is Option E.

The external ear provides a versatile cartilage source for reconstructive procedures, especially for augmentative rhinoplasty. Short- and long-term morbidity associated with ear cartilage harvest using concha, tragus, and scapha as donor sites were evaluated. The relevant morbidity factors in the early postoperative period were hematoma formation (6.7%) and sensory impairment (3.3%). In long-term follow-up, sensory impairment was the most frequent condition objectively assessed and subjectively complained of (12.9%). Overall, sensory impairment was confined to concha as the donor site. Anthropometric measurements showed a mean difference in the length of the affected ear compared with the contralateral ear of 1.8 mm, a width difference of 2.5 mm, a difference in tragus/lateral canthus distance of 1.4 mm, and a difference in protrusion angle of 2.4 degrees. Aesthetically relevant complications were rare and their occurrence restricted to single cases. Long-term cases of necrosis, infection, and perichondritis were exceedingly rare. Cartilage graft harvest from the auricle can be considered as a relatively safe procedure with a favorable aesthetic outcome. Cauliflower ear may result from long-standing loss of blood supply to the ear cartilage and formation of neocartilage from disrupted perichondrium. This complication is almost never seen from harvest but from auricular trauma.

47
Q

hos73A 21-month-old male infant is scheduled to undergo reconstruction of a 12-cm² cranial defect after a cranial vault reshaping procedure. Which of the following materials is most appropriate for cranioplasty in this patient?

A ) Bone dust
B ) Hydroxyapatite
C ) Particulate bone
D ) Split calvarium
E ) Split rib

A

The correct response is Option C.

Although alloplastic materials and bone substitutes have been used to reconstruct the pediatric cranium, autogenous bone is the best option. Unlike synthetic materials, autogenous grafts undergo rapid osseointegration because they are osteogenic, osteoinductive, and osteoconductive. Once healed, the grafted site has the same properties as native bone: strength, growth potential, stability, and resistance to infection.

Compared with other sources of autogenous graft, calvarial bone is the preferred donor site for cranioplasty, especially in pediatric patients, because it is in the operative field, volume retention and strength of the graft are excellent, and harvest causes no additional discomfort. Extracranial sources of autologous bone (rib or iliac crest) provide little volume in toddlers, have a higher resorptive rate than cranial bone, and are associated with greater donor-site morbidity.

Bone dust is the powder-like bone that results from using a high-speed burr. Bone dust is ineffective and resorbs when used for cranioplasty, possibly because of its smaller particle size and/or thermal injury during harvesting. Autogenous bone, the standard material used for cranial reconstruction, is of limited supply in young children. Calvarial bone is difficult to split until a diploic space has formed, usually after 4 years of age, and some authors do not recommend in situ harvest before 9 years of age. Cranial particulate bone grafting, however, can be harvested at any age, and studies have demonstrated that it is as effective as split calvarial bone graft for closure of cranial defects. Particulate cranial graft consists of tiny pieces of bone harvested with a low-speed, hand-driven bit and brace. It has shown in animal models as well as clinical studies to effectively heal full-thickness cranial defects.

48
Q

A 42-year-old woman with a history of progressive facial atrophy comes to the office because of a moderately sized soft-tissue deficit on the left side of the face. She is scheduled to undergo a single fat transfer procedure for correction. Which of the following is the most likely outcome of this procedure in this patient?

A ) Donor site seroma
B ) Facial nerve injury
C ) Fat embolism
D ) Hypertrophic scarring
E ) Inadequate correction

A

The correct response is Option E.

Romberg disease, or progressive facial atrophy, is a rare pathologic process characterized by an acquired, idiopathic, self-limited, unilateral atrophy of the face, variably involving skin, subcutaneous tissues, fat, muscle, and less frequently, the underlying bone structures. Methods of restoration of facial contour and volume in these patients include synthetic implants, bone grafts, free tissue transfer, and fat grafting.

Fat injections have been used for over 20 years to correct soft-tissue deformities throughout the body. Fat grafts are often used for ?touch-up? of various reconstructive procedures. This technique has been found to be helpful as an adjunct to free tissue transfer in cases of progressive facial atrophy and congenital hemifacial microsomia.

Although fat embolism has been rarely reported during fat transfer in the face, complication rates of fat grafting are low, especially when compared with complications of free tissue transfer for reconstruction of facial deformities, which may include donor site seroma, facial nerve injury, and hypertrophic scarring. The most common adverse outcome of fat grafting is likely to be inadequate correction with a single-stage procedure. A recent study found that no statistically significant difference was found in satisfaction rates between free flap reconstruction and serial fat grafting of soft-tissue deficits in hemifacial microsomia.

49
Q

A 10-year-old boy comes to the office because of difficulty turning his head and looking up 2 years after sustaining burns in a gasoline fire involving nearly 50% of the total body surface area. Bilateral skin grafting of the distal upper extremities and proximal upper thighs was performed 2 years ago to treat deep partial-thickness burns. Current physical examination shows severe burn scar contracture on the neck (shown) and healed skin grafting of the neck and anterior trunk. After release and excision of the burn scar contracture, a 10 × 20-cm defect results. Which of the following is the most appropriate method of reconstruction to provide the most aesthetic and functional outcome?

A ) Anterolateral thigh perforator free tissue transfer to the neck
B ) Dermal regeneration template (Integra)
C ) Full-thickness skin graft from the inguinal region
D ) Split-thickness skin graft from the back
E ) Split-thickness skin graft from the posterior thigh

A

The correct response is Option B.

While Integra artificial skin substitute has been used traditionally in the acute management of burns, there have been many successful reports incorporating its use in general reconstructive plastic surgery. Integra is a bioengineered dermal substitute consisting of a bilayer membrane system. The dermal replacement layer is composed of a cross-linked bovine tendon collagen-glycosaminoglycan (chondroitin-6-sulfate) matrix coated on one side with a synthetic polysiloxane polymer (silicone) layer. Integra “take” mirrors skin graft “take” and follows the sequence of (1) imbibition; (2) fibroblast migration; (3) neovascularization; and (4) maturation.

As the host tissue infiltrates the dermal layer, the collagen layer is biointegrated with the wound to form a vascular neodermis capable of accepting a split-thickness skin graft following a period of 3 to 4 weeks. Proper patient selection and careful monitoring for infection or hematoma accumulation are crucial in this time period. After approximately 4 weeks, the silicone layer is removed and a thin (0.008- to 0.010-in) skin graft can be applied.

Advantages of Integra use include improved cosmesis; diminished burn scar contracture or development of hypertrophic scar; off-the-shelf availability in large quantities; increased elasticity; and the ability to use thinner skin grafts with subsequent improvement in donor site morbidity, scar, and time to heal. Other cited advantages include commensurate growth of the grafted tissue in children, potential avoidance of a microsurgical procedure, and placement over tendons with no significant reduction in tendon mobility. The disadvantages of Integra include its high cost, steep learning curve, need for at least two stages with subsequent increased time to final closure, potential for infection under the silicone layer, and lack of adnexal structures with patient complaint of skin dryness.

In the patient described, use of Integra with delayed, thin split-thickness skin grafts at 4 weeks will give the best outcome in terms of cosmesis and function. Free tissue transfer is an excellent technique for resurfacing the contracted neck following burns, but the thigh donor site has already been treated with skin grafts in the scenario described, and the aesthetic result would be suboptimal. A better choice would be the scapular or parascapular donor site, perhaps in combination with a preliminary tissue expansion of that location. Coverage with a full-thickness skin graft would diminish the risk of secondary contraction; however, harvesting enough tissue to cover the entire neck would be difficult with the inguinal region as a donor site. Coverage with a split-thickness skin graft is not an appropriate option in this location because of the risk of secondary contraction and recurrence of the neck contracture.

50
Q

A 9-year-old girl with a unilateral cleft lip and palate undergoes alveolar bone grafting with a cancellous iliac graft. Which of the following is the most likely mechanism by which the bone graft will heal in this patient?

A ) Endochondral ossification
B ) Osteoconduction
C ) Osteogenesis
D ) Osteoinduction
E ) Progenitor cell recruitment

A

The correct response is Option C.

The most likely mechanism of cancellous bone graft healing is by osteogenesis. Cancellous and vascularized bone grafts heal primarily by osteogenesis. Because these grafts are rapidly revascularized, osteoblasts survive the transplantation and produce new bone at the recipient site.

Endochondral ossification is the embryologic process by which bones of the appendicular skeleton, vertebral column, and skull base are formed. The maxilla develops by membranous ossification.

Osteoconduction, or “creeping substitution,” is the mechanism by which cortical bone grafts heal (e.g., split cranial bone graft). After cortical bone is separated from its blood supply, it serves as a nonviable scaffold for the ingrowth of blood vessels and osteoprogenitor cells from the recipient site. This leads to resorption and replacement of most of the graft with new bone. The graft becomes fully osseointegrated with the recipient site.

Osteoinduction involves the stimulation of mesenchymal cells at the recipient site to differentiate into bone-producing cells. Demineralized bone and bone-morphogenic protein produce new bone by osteoinduction.

Although progenitor cells might be recruited to an area where bone grafting has occurred, they are not a primary mechanism for cancellous bone graft healing.

51
Q

A 34-year-old woman is referred for evaluation because of a 9 × 7-cm lateral soft-tissue deformity of the right thigh with skin thickening. She has a history of oncologic resection and external beam radiation of 50 Gy. Which of the following is the most appropriate step in management?

A ) Acellular dermis
B ) Dermal-fat grafting
C ) Full-thickness skin grafting
D ) Lipoaspirate injection

A

The correct response is Option D.

Radiation has deleterious effects on local vascularity, fibroblast activity, growth factor levels, and mesenchymal stem cell populations. Microscopic examination of irradiated tissue shows microvascular thrombosis and abnormal vasculature. Clinically, irradiated wounds are associated with slower epithelialization, decreased tensile strength, and higher infection and dehiscence rates. Although data are still limited, marked improvements in irradiated tissues have been reported with autologous fat injection. Lipoaspirate injected in areas of chronic radiation wounds improved the wound quality by either promoting a more vascular wound amenable to reconstruction or spontaneous closure. Adipocytes contain stem cells which improve wound vascularity.

Acellular allogeneic dermis is produced from human cadaveric allograft skin. During the skin processing, immunogenic components that include all viable cells are extracted, leaving acellular dermis and extracellular matrix intact. The entire epidermis and all of the dermal cells are removed during a freeze-drying process. The resultant matrix has undamaged collagen types IV and VII, elastin, and laminin. The acellular and porous dermal matrix allows ingrowth and colonization by host fibroblasts and endothelial cells. Acellular allogeneic dermis provides a template for fibrous ingrowth, resulting in an integrated graft that is not rejected. It can be used for almost any area as long as there is adequate blood supply to support the graft. Acellular allogeneic dermis survival in less vascular areas is unknown. Dermal-fat grafts are usually harvested from the same areas as full-thickness skin grafts are. These include the lower abdomen, the suprapubic or periumbilical regions, the gluteal or inframammary folds, the subiliac crest, and even the forearm for hand surgery purposes. An ellipse of skin with dimensions appropriate to fill the recipient defect is outlined. The epithelium can be removed by sharp excision with a scalpel or dermabrasion. Previous radiation, excessive cicatrization, deficiency of circulation, and poor healing qualities caused by poor nutrition and systemic diseases all militate against a satisfactory result.

Skin grafts are used in a variety of clinical situations. The essential indication for the application of a skin graft is wound closure. In general, full-thickness skin grafts are applied to the regions of the face, ears, and hands. Split-thickness skin grafts are usually placed on the trunk and genitalia. Skin grafts are usually the initial treatment of choice for many open wounds that cannot be closed primarily. Grafting offers the simplest method of wound closure in the reconstructive ladder, assuming that primary closure is not possible or would lead to undue tension. Skin grafts are generally avoided in management of more complex wounds. Conditions with deep spaces and exposed bones, such as open sternal wounds, pressure sores, and open fractures, normally require the use of skin flaps or muscle flaps for stable wound coverage. Skin grafts have limited success in wounds with a compromised blood supply, such as irradiated wounds and ischemic ulcers.

52
Q

A 34-year-old woman is referred for evaluation because of a 9 × 7-cm lateral soft-tissue deformity of the right thigh with skin thickening. She has a history of oncologic resection and external beam radiation of 50 Gy. Which of the following is the most appropriate step in management?

A ) Acellular dermis
B ) Dermal-fat grafting
C ) Full-thickness skin grafting
D ) Lipoaspirate injection

A

The correct response is Option D.

Radiation has deleterious effects on local vascularity, fibroblast activity, growth factor levels, and mesenchymal stem cell populations. Microscopic examination of irradiated tissue shows microvascular thrombosis and abnormal vasculature. Clinically, irradiated wounds are associated with slower epithelialization, decreased tensile strength, and higher infection and dehiscence rates. Although data are still limited, marked improvements in irradiated tissues have been reported with autologous fat injection. Lipoaspirate injected in areas of chronic radiation wounds improved the wound quality by either promoting a more vascular wound amenable to reconstruction or spontaneous closure. Adipocytes contain stem cells which improve wound vascularity.

Acellular allogeneic dermis is produced from human cadaveric allograft skin. During the skin processing, immunogenic components that include all viable cells are extracted, leaving acellular dermis and extracellular matrix intact. The entire epidermis and all of the dermal cells are removed during a freeze-drying process. The resultant matrix has undamaged collagen types IV and VII, elastin, and laminin. The acellular and porous dermal matrix allows ingrowth and colonization by host fibroblasts and endothelial cells. Acellular allogeneic dermis provides a template for fibrous ingrowth, resulting in an integrated graft that is not rejected. It can be used for almost any area as long as there is adequate blood supply to support the graft. Acellular allogeneic dermis survival in less vascular areas is unknown. Dermal-fat grafts are usually harvested from the same areas as full-thickness skin grafts are. These include the lower abdomen, the suprapubic or periumbilical regions, the gluteal or inframammary folds, the subiliac crest, and even the forearm for hand surgery purposes. An ellipse of skin with dimensions appropriate to fill the recipient defect is outlined. The epithelium can be removed by sharp excision with a scalpel or dermabrasion. Previous radiation, excessive cicatrization, deficiency of circulation, and poor healing qualities caused by poor nutrition and systemic diseases all militate against a satisfactory result.

Skin grafts are used in a variety of clinical situations. The essential indication for the application of a skin graft is wound closure. In general, full-thickness skin grafts are applied to the regions of the face, ears, and hands. Split-thickness skin grafts are usually placed on the trunk and genitalia. Skin grafts are usually the initial treatment of choice for many open wounds that cannot be closed primarily. Grafting offers the simplest method of wound closure in the reconstructive ladder, assuming that primary closure is not possible or would lead to undue tension. Skin grafts are generally avoided in management of more complex wounds. Conditions with deep spaces and exposed bones, such as open sternal wounds, pressure sores, and open fractures, normally require the use of skin flaps or muscle flaps for stable wound coverage. Skin grafts have limited success in wounds with a compromised blood supply, such as irradiated wounds and ischemic ulcers.

53
Q

A 27-year-old man is scheduled to undergo excision and skin grafting after sustaining a full-thickness burn to the dorsum of the hand. To optimize graft take in this patient, which of the following is the most important aspect of management?

A ) Meshing of the skin graft
B ) Meticulous hemostasis
C ) Negative pressure wound therapy
D ) Use of fibrin glue
E ) Use of a thin split-thickness skin graft

A

The correct response is Option B.

The most important aspect of recipient site management to optimize graft survival and outcome in this patient undergoing dorsal hand skin grafting is meticulous hemostasis. This is because hematoma is the leading cause of skin graft loss. Meshing a skin graft may promote graft survival by providing a mode of egress for blood and seroma that might form below the graft. However, it is associated with an increased rate of secondary contraction and unfavorable cosmetic results. Both of these factors make meshing undesirable when grafting the dorsum of the hand.

Use of a negative pressure wound therapy dressing can result in improved graft survival, especially in recipient sites with irregular contours. However, it is not critical for the hand. A good dressing and proper immobilization should achieve the same result. Use of this type of dressing is not as important as meticulous hemostasis for ensuring graft survival and outcome.

Fibrin glue has been used to promote graft survival by improving hemostasis and graft adherence. There is also some evidence that fibrin glue may inhibit wound contraction. Nevertheless, it is no substitute for meticulous hemostasis and should only be used as an adjunct to the fundamental techniques of skin graft placement.

Use of a thin split-thickness skin graft is associated with improved graft survival when compared with a thick split-thickness or full-thickness skin graft, but it is also associated with an increased rate of secondary contraction that is not desirable on the dorsum of the hand. Balancing the need for graft survival versus the need to avoid secondary contraction is an important consideration in burn surgery. Using thinner grafts will optimize graft survival, but this is not as critical as ensuring good hemostasis.

54
Q

A 63-year-old woman is scheduled to undergo autologous fat injection to improve the contour and increase the size of the right breast. She underwent reconstruction of the right breast with a latissimus dorsi flap 10 months ago because of mastectomy. The patient does not have or desire a breast prosthesis. Which of the following is the most likely sequela of autologous fat injection in this patient?

A) Calcification
B) Donor site irregularity
C) Fat resorption
D) Hypertrophic scarring
E) Skin necrosis

A

The correct response is Option C.

Although controversial, autologous fat injection (lipo-modeling) to the breast and the reconstructed breast has gained popularity in recent years. This can be attributed to several factors, including the publication of large numbers of patient series’ demonstrating the safety, efficacy, and improvements in the harvest and preparation of fat. However, the most common complication of fat injection remains to be the resorption of the grafted fat, ranging from 30 to 70%.

Fat and skin necrosis, calcification, hypertrophic scarring, and contour irregularity of both the recipient and donor sites can occur, but to a lesser extent. The rate of skin necrosis is low. Hypertrophic scarring is more common in patients with a history of poor scarring. In the case of calcifications, pre- and postoperative examination by a radiologist specialized in breast imaging is necessary to limit the risk of breast cancer, which may occur coincidentally with lipo-modeling.

55
Q

A 33-year-old man who sustained burns to 95% of the total body surface area five days ago is scheduled to undergo the initial stages of surgical reconstruction. In preparing this patient, which of the following is the advantage of using cultured epidermal autografts versus split-thickness skin grafts?

A ) Donor site

B ) Durability

C ) Elasticity

D ) Immediate availability

E ) Reduced expense

A

The correct response is Option A.

With use of cultured epidermal autografts, no donor site limitations exist. The patient €™s own keratinocytes are expanded in tissue culture and a small skin specimen may be cultured and expanded within two to three weeks. Unfortunately, there is no dermal matrix tissue and, therefore, the graft lacks the elastic quality of normal skin or even split-thickness skin grafts. This results in wounds that are stiff, and motion is limited in the face and around joints. Likewise, the lack of a dermis results in very slow basement membrane formation; therefore, there are frequent problems with blistering and easy shearing. The use of cultured epidermal autografts is somewhat limited by its high cost and delay in availability as the tissue is cultured.

56
Q

A 34-year-old man undergoes correction of the defect shown three years after sustaining an injury to the left tip of the nose while playing football. Placement of an alar batten graft is planned. During septal graft harvest, the mucoperichondrial plane is difficult to elevate, and the cartilage is removed with an adherent perichondrial layer. Which of the following is most likely to result from the use of this graft compared with a cartilage-only graft?

A ) Extrusion

B ) Necrosis

C ) Ossification

D ) Resorption

E ) Warping

A

The correct response is Option E.

Pure cartilage grafts tend to maintain shape, but grafts with an intact perichondrial layer can curl significantly and lead to unpredictable results. During septal graft harvest, care must be taken to elevate mucoperichondrial flaps in the proper plane. Likewise, auricular or costal cartilage grafts must be harvested in a subperichondrial plane. Removal of the perichondrium and softer outer cartilage layer leaves the more rigid cartilage core, which maintains shape more predictably.

Extrusion, necrosis, ossification, and resorption are not known to be affected by the presence or absence of the perichondrial layer.

The alar batten graft is a useful means of adding support to a deformed or weakened alar cartilage

Warped cartilage has a €œmemory € and tends to return to its warped shape unless adequate support is provided.

57
Q

A 51-year-old woman has loss of vision in her left eye immediately after autogenous fat injections to the face and nasojugal regions performed under local anesthesia with 2 mL of 1% lidocaine with 1:100,000 epinephrine. The procedure was performed using small boluses of fat, which were injected slowly into the tear trough. Physical examination shows loss of vision in the left eye. Which of the following occurrences is the most likely cause of this complication?

A ) Fat embolism

B ) Glaucoma

C ) Lidocaine toxicity

D ) Retrobulbar hematoma

E ) Vasovagal response

A

The correct response is Option A.

Blindness and strokes have occurred as a result of the injection of soft-tissue fillers in almost every part of the face: glabella, forehead creases, temple, nose, cheeks, nasolabial folds, and lower lip. The injection of large boluses of soft-tissue fillers in the face and the use of needles or cannulas that can easily perforate an arterial wall should be avoided. Fat injections into the face lead to an acute local increase in pressure in highly vascularized tissue. Fragments of fatty tissue reach ocular and cerebral arteries by reversed flow through branches of the carotid arteries after they are introduced into facial vessels. The manifestation of fat embolism appears either immediately after the fat injection or after a latency period. To minimize the risk of such a major complication, fat injections should be performed slowly, with the lowest possible force. Fat injections into pretraumatized soft tissue, for example, after rhytidectomy, should be avoided because of the increased risk of intravasation of fat particles.

58
Q

After harvesting of a split-thickness skin graft from the lateral aspect of the thigh, application of which of the following types of wound care agents will yield the most rapid epithelialization at the donor site?

(A) Normal saline wet-to-dry gauze

(B) Occlusive dressing

(C) Petrolatum-impregnated gauze

(D) Semi-occlusive dressing

(E) Silver sulfadiazine cream

A

The correct response is Option D.

The ideal donor site dressing is one that promotes rapid re €‘epithelialization, causes little pain, requires little care, is inexpensive, and has a low rate of infection. Options include occlusive dressings (Duoderm), semi €‘occlusive dressings (OpSite, Tegaderm), semi €‘open dressings (petrolatum-impregnated gauze, Xeroform or scarlet red), and no dressing.

Although semi €‘open techniques using a heat lamp to dry the donor site covered with Xeroform or scarlet red are historically popular, these dressings do not meet current concepts of promoting wound healing by providing a moist environment.

In multiple studies, the superior dressings have been shown to be semi €‘occlusive. These products have been shown to have the fastest healing rates (average nine days to re €‘epithelialization), lowest subjective pain scores, lowest infection rates (~3%), and are among the lowest in cost. They have the advantage of being transparent, which allows ongoing inspection of the site while maintaining sterility. Some fluid collects under these materials, which promotes moist wound healing and probably accounts for the more rapid healing rates and decreased subjective pain scores.

Donor sites for split €‘thickness skin grafts heal spontaneously from epithelial cells remaining in epithelial appendages within the dermis and at the wound edges. Healing begins within 24 hours of harvesting, and the rate of healing is directly proportional to the number of epithelial appendages remaining and inversely proportional to the thickness of graft harvested. When the epidermis has regenerated it may be reharvested; however, each harvesting removes a portion of dermis that is not regenerated. The initial epithelium that is regenerated is very delicate and easy to disrupt with tape or dressing changes. This is another reason to use the semi €‘occlusive dressing technique that does not need to be removed until healing is complete.

59
Q

A 30-year-old man undergoes reconstruction of the right lower leg after sustaining an open fracture of the tibia. A skin graft is harvested and placed over a free muscle flap. Two days postoperatively, which of the following findings on microscopic examination of the skin graft is most likely?

(A) Early ischemic injury

(B) Increased collagen cross €‘linking

(C) Neovascular circulation

(D) Significant edema

(E) Venous congestion

A

The correct response is Option D.

Within the first 24 hours after placement, the graft survives by serum imbibition, which is absorption of nutrients from the serum leaked from the donor site (muscle in this case). At 24 hours, the healing graft will have increased in mass from edema by up to 30%. Leukocytes can be seen invading the graft, which may help stimulate endothelial migration and revascularization. By 24 hours, donor site vessels have begun to invade the graft vascular channels in a process called inosculation. The graft vessels degenerate and become replaced by the growing donor site vessels. Circulation is reestablished by day 4 to 5 at the earliest. Whereas early graft ischemia results in a lowering of the pH and a decrease in metabolism, ischemic injury and necrosis are not seen. Graft maturation and collagen turnover occur over weeks to months. Early recognition and correction of the venous thrombosis are not likely to affect early graft nourishment and healing.

60
Q

A 27-year-old woman is scheduled to undergo rhinoplasty using homograft rib cartilage for reconstruction of the dorsum of the nose. Which of the following interventions during this procedure is most effective to reduce long €‘term warping of the graft?

(A) Access to a peripheral segment

(B) Insertion of the graft at least 30 minutes after carving

(C) Scoring of the graft

(D) Suture fixation

(E) Use of nonirradiated material

A

The correct response is Option B.

To minimize the long €‘term clinical effects of cartilage warping, it is recommended to wait at least 30 minutes after carving the graft to allow initial warping to occur. The observed warping can then be accounted for in the final graft placement. Further warping may continue for some time, but the majority will occur within the first 30 to 60 minutes.

Use of nonirradiated graft material may predispose more warping than irradiated material; however, this may depend on the dose of radiation. Doses of 3 to 4 million rads are less likely to result in graft warping compared with no radiation, but at doses of 1.5 to 2.5 million rads, the warping may be similar to that of nonirradiated grafts.

Central cuts of cartilage grafts are less likely to warp than are peripheral cuts. Scoring of the graft will result in warping of the graft. Suture fixation will not prevent graft warping.

61
Q

Compared with cortical bone, which of the following best characterizes autologous cancellous bone grafts?

(A) Effective in bridging defects larger than 6 cm

(B) Greater structural strength

(C) Less osteoconductive

(D) Less osteoinductive

(E) More readily revascularized and remodeled

A

The correct response is Option E.

Relative to cortical bone, cancellous bone grafts are more osteoconductive (the property of the scaffold €‘like matrix to accommodate the ingrowth of new bone) and more osteoinductive (the capacity to induce mesenchymal cells from the recipient bed to produce active osteoblasts). Cancellous bone is more quickly revascularized, which usually occurs within two weeks of grafting, whereas cortical bone can take up to two months to revascularize. Cancellous grafts are also more easily remodeled. Cancellous bone grafts are ideal for bridging bone gaps of less than 5 to 6 cm, but suffer from a lack of structural rigidity until 6 to 12 months after grafting, when they are generally as strong as cortical bone grafts.

62
Q
  • Under average conditions, which of the following percentages represents the expected six-month resorption rate of fat injections?
  • (A) 10%
  • (B) 30%
  • (C) 50%
  • (D) 70%
  • (E) 90%
A

Please note: Question 4 has been deleted from the examination.

The key points of the theory of fat survival are as follows: 1) fat is a dynamic tissue; 2) cells that suffer trauma lose more volume; and 3) the recipient site makes an important difference in graft survival based on the vascularity of the bed. Careful handling of the graft is critical. Therefore, given the ideal conditions of minimal traumatic technique, meticulous fat graft size, and recipient bed selection, we should expect approximately 60% to 80% long-term graft survival based on several studies.

Moore and co-workers reported on the effects of mechanical damage to fat cells harvested with syringe suction lipectomy versus excision with local anesthetics. The mechanical damage to tissue aspirated versus tissue that was excised was unchanged. Lidocaine inhibited the glucose metabolism and lipolysis of adipocytes in culture. The effect was maintained only for as long as the lidocaine was present.

Also, as Smahel reported, fat grafts, like other grafts, must be small enough to be revascularized but large enough to maintain structural integrity and some native blood vessels. When part of the graft is not revascularized, the fat becomes necrotic and is not simply resorbed but is broken up and removed by the cellular elements.

Karaoglu and co-workers have studied the recipient site of fat grafts and seen a statistically significant increase in fat graft survival in supramuscular layer (81.95 “ 4.40%) than in subcutaneous (41.62 “ 3.29%) and submuscular layer (37.31 “ 5.77%) (P < 0.05). This study demonstrates that selection of an appropriate recipient site should enhance ultimate survival of the fat graft.

63
Q

A 26-year-old woman of Asian descent who underwent rhinoplasty five years ago has erosion of the silicone rubber (Silastic) prosthesis through the skin of the nasal tip. Physical examination shows a depressed scar in this region. Secondary rhinoplasty is planned. Use of which of the following grafts for this procedure is most appropriate to minimize volume loss?

(A) Dermis

(B) Fat

(C) Muscle

(D) Cartilage

(E) Bone

A

The correct response is Option D.

A cartilaginous graft would be most appropriate to correct the deformity described. The low metabolic rate of cartilage leads to minimal volume loss. Fat graft survival depends on early neovascularization and only approximately 50% of lipocytes in an autogenous graft survive. Owing to the traumatic nature of the suction technique, fewer viable micrografts are transferred with potential for survival so that surgically removed grafts (macrografts) have a greater longevity. Animal studies have also demonstrated that adipocytes implanted in a vascularized bed (muscle) survive better than those in dermis. Considerable resorption is seen in nonvascularized bone grafts. Muscle alone is not routinely transferred as a graft. There are not sufficient studies in the literature to support the routine use of dermis to correct this deformity.

64
Q

A 45-year-old woman undergoes placement of a dorsal only cartilage graft during secondary rhinoplasty. Which of the following operative steps is most likely to minimize warping of the cartilage graft?
(A) External stabilization
(B) Maintaining recipient bed vascularity
(C) Precise graft fixation
(D) Preservation of perichondrium
(E) Symmetrical design

A

The correct response is Option E.

Autogenous rib cartilage grafts have gained more widespread use in rhinoplasty as dorsal onlay grafts and columellar struts. However, the usefulness of rib as a donor site has been limited by difficulties with postoperative cartilage warping.

Symmetrical graft design decreases cartilage warping because it regulates the tendency of the inner mass of cartilage to expand and become distorted during the harvesting process. Studies have shown that the compact outer subperichondrial layer regulated the tendency of a loose inner cartilaginous mass to warp. Symmetric graft design was found to more evenly regulate the stresses that caused warping. When considering donor sites for the nasal dorsum, costal cartilage ribs have been shown to be particularly useful because of the availability of a generous and naturally straight segment requiring minimal trimming and carving.

Internal stabilization of autogenous rib cartilage grafts with Kirschner wires has been found to effectively prevent graft warpage.

Preservation of graft perichondrium influences cartilaginous growth through the preservation of chondrocytes. This concept has been found to be of particular use in the fabrication and reconstruction of ear cartilage framework in children.

Precise graft fixation is important in the avoidance of external distortion due to graft migration or displacement.

Maintaining recipient bed vascularity is important to avoid graft extrusion resulting from tissue compromise.

65
Q

Acellular dermal matrix (AlloDerm) is used in plastic surgery for each of the following purposes EXCEPT
(A) correcting retraction of the lower eyelid after blepharoplasty
(B) decreasing adhesions after repair of an abdominal hernia
(C) reducing incidence of oronasal fistulas after cleft palate repair
(D) resurfacing thin pockets in mammaplasty
(E) treating osteomyelitis of the tibia

A

The correct response is Option E.

AlloDerm is cadaveric dermis that is processed to be acellular and nonimmunogenic and is then freeze-dried for preservation. Although AlloDerm seems to be used ubiquitously in plastic surgery; it is not indicated for management of osteomyelitis. One of the principles in management of osteomyelitis is the transfer of healthy vascularized tissue. Although AlloDerm provides a substrate for tissue ingrowth, it does not bring in any blood supply.

Use of AlloDerm matrix has been described in the literature for abdominal wall reconstruction, lower eyelid reconstruction, lining of breast implant capsules, and cleft palate repair.

66
Q

Which of the following is the best donor site for delayed multiple harvesting of split-thickness skin grafts?

(A) Back
(B) Lateral forearm
(C) Medial arm
(D) Medial forearm
(E) Medial thigh

A

The correct response is Option A.

The selection of a donor site depends largely on donor site morbidity and skin thickness. The back provides a nearly ideal donor site for repeated harvesting of split-thickness skin grafts and has large areas of thick skin available for harvesting. The lateral forearm exhibits unacceptable donor site morbidity. The medial arm, medial forearm, and medial thigh have skin of insufficient thickness to allow multiple harvesting.

The number of times that a donor site can be harvested for split-thickness skin grafts is limited by the thickness of the dermis at the site. A split-thickness skin graft includes the epidermis and part of the dermis. The donor site of a split-thickness graft heals by migration from the remnant epithelia of the dermal appendages, such as hair roots and sweat and sebaceous glands. Therefore, the epidermis regenerates but the dermis does not. A repeat split-thickness graft may be harvested once the skin has reepithelialized, but a thinner dermis will remain at the donor site.

67
Q

Which of the following bone grafts exhibits the greatest inductive capacity?

(A) Allogenic
(B) Autologous cancellous
(C) Autologous cortical
(D) Free vascularized
(E) Xenogenic

A

The correct response is Option B.

Cancellous bone grafts have the greatest inductive capacity (ability to stimulate the formation of new bone) because they contain bone morphogenic proteins that stimulate bone growth. Cortical bone grafts and allogenic and xenogenic grafts have less inductive capacity. Free vascularized bone grafts have no inductive capacity because they do not rely on stimulating new bone formation.

68
Q

Which of the following is an advantage of using the dermal regeneration template (Integra) instead of a thin split-thickness autograft for reconstruction of the hand?

(A) Elimination of donor site
(B) Improved cosmesis
(C) Increased wound contraction
(D) Reduced risk of hematoma
(E) Shorter healing time

A

The correct response is Option B.

The advantages of Integra include the availability of large quantities, the simplicity and reliability of the placement technique, and the pliability and cosmetic appearance of the resulting cover.
Integra does not eliminate the use of a donor site. However, the skin graft used at the second stage is typically thinner than that used in single-stage grafting, so Integra does avoid the use of a deep donor site, which decreases the risk of infection, scarring, and permanent pigment changes.

Wound contraction with Integra is typically less than that with single-stage split-thickness skin grafting. Compared with autologous skin, Integra purportedly has no decrease in the hematoma rate. In one study, the incidence of hematoma under Integra was 9 out of 39 cases. Because Integra requires a second surgery after 3 to 4 weeks for coverage with a thin split-thickness skin graft, it has a longer healing time until final wound coverage.

69
Q

Which of the following grafts has minimal resorption and loss of volume?

(A) Bone
(B) Cartilage
(C) Macro-fat
(D) Micro-fat
(E) Muscle

A

The correct response is Option B.
Although all of these tissues may be used for grafts, autologous cartilage grafts offer minimal resorption and loss of volume and provide good results in many clinical settings. The metabolic rate of cartilage is 1/100 to 1/500 the rate of other human tissues. It has low glycolytic activity and consumption because of its small cell population and relative isolation by the cartilage matrix. This isolation helps protect cartilagenous tissue from resorption. Readily accessible sites for cartilage grafts are the septum, auricular cartilage, and ribs.

Bone grafts, which depend on the size of the bone, have variable resorption rates based on the clinical circumstance. Macro-fat grafts are usually unreliable. They have high resorption rates and are more effectively used as dermal fat grafts to minimize resorption and fibrosis. Given by lipoinjection, micro-fat grafts may exhibit greater longevity than macro-fat grafts. However, their resorption may be 25% to 50% of the injected volume. Muscle grafts are not routinely used. Rather, vascularized muscle flap grafts are preferred for transferring muscle tissue.

70
Q

Which of the following bone grafts does NOT rely on creeping substitution as a mode of remodeling?

(A) Allogenic
(B) Autologous cancellous
(C) Autologous cortical
(D) Free vascularized
(E) Xenogenic

A

The correct response is Option D.

Free vascularized bone grafts do not rely on creeping substitution (replacement of necrotic bone with osteoblasts and new vascular ingrowth) for remodeling. They do not need to stimulate new bone formation because they are used when little or no bone has been lost.

All nonvascularized bone grafts undergo a degree of resorption and remodeling, including creeping substitution, and have some degree of inductive capacity (ability to stimulate new bone formation). Cancellous bone grafts contain bone morphogenic proteins that stimulate the formation of new bone. Cortical bone grafts and allogenic and xenogenic grafts have less of this inductive capacity.

71
Q

Which of the following biologic dressings contains only human-derived materials?

(A) Alloderm
(B) Apligraf
(C) Biobrane
(D) Integra
(E) Transcyte

A

The correct response is Option A.

Each of the biologic dressings listed above is either a synthetic or tissue-engineered biologic skin substitute that will provide a clean, moist environment and protect the open wound until definite wound coverage is obtained.

Alloderm is lyophilized acellular human cadaveric dermis. This dressing contains only human-derived materials. Because it is devoid of immunogenic cells, it does not undergo the rejection that occurs with cadaveric skin. Alloderm can also be used as soft-tissue filler in patients undergoing cosmetic procedures. In some instances, thin layers of this dressing can be placed on a wound bed to vascularize and support a skin graft.

Apligraf is comprised of a bilayer of neonatal epidermal keratinocytes and dermal fibroblasts within a matrix of bovine collagen. This dressing is used for coverage of venous stasis ulcers and foot ulcers. Its mechanism of action involves graft adherence and subsequent remodeling on the wound bed.
Biobrane is a synthetic wound dressing composed of nylon and silicone fabric coated with porcine collagen. It provides temporary coverage of partial-thickness wounds and can be removed when epithelialization of the wound occurs.

Integra is a bioengineered dermal substitute that consists of a dermal matrix of bovine collagen and shark-derived chondroitin-6-sulfate covered by a bilayer of Silastic epidermis. When the dermal replacement layer becomes infiltrated with capillaries, fibroblasts, and inflammatory cells, the Silastic cover can be removed and autografting can be performed.

Transcyte is a dermal substitute comprised of neonatal dermal fibroblasts cultured onto a thin, semipermeable membrane of silicone that is bonded to a nylon mesh and bovine collagen. The fibroblasts secrete structural proteins and growth factors, creating a dermal matrix for future grafting or temporary coverage of partial-thickness burns.

72
Q

Which of the following is the primary advantage of using Integra (artificial skin) for coverage of full-thickness burns?

(A) Allowing for immediate use of cultured epithelial autografts
(B) Allowing for use of thinner autografts
(C) Avoidance of autografting
(D) Complete revascularization within seven days
(E) Prevention of fibroblast ingrowth into the dermal replacement layer

A

The correct response is Option B.

Integra, or artificial skin, is the most widely used skin substitute. It is a synthetic bilaminar membrane composed of a dermal matrix of porous bovine collagen cross-linked with shark-derived chondroitin-6-sulfate and covered by temporary Silastic epidermis. It does not contain neonatal fibroblasts or autologous keratinocytes. A two-stage application process is required; the initial stage is application of the dressing.

Following its application, the dermal matrix of the Integra dressing acts as a template, becoming infiltrated with host fibroblasts, endothelial cells, and inflammatory cells. The host collagen gradually replaces the bovine collagen during the healing process, and the silicone cover controls moisture loss and protects the wound. Adequate revascularization occurs within two to three weeks, at which time the superficial silicone layer will have sloughed off as a result of ingrowth through the collagen and glycosaminoglycan matrix. At this time, the Integra graft can be removed, and a thinner autograft, such as a very thin sheet of split-thickness skin graft, can be applied for durable coverage.

In addition to allowing for use of thinner autografts, advantages of Integra include immediate temporary coverage, more rapid healing of donor sites, and improved cosmesis.

Because the revascularization process is delayed, cultured epithelial autografts would fail to take if applied immediately because of the lack of vascularization in the wound bed.

Although Integra can be used to cover partial-thickness burns, it does not relieve the need for autografting over full-thickness burns.

As mentioned above, Integra is revascularized in two to three weeks, not within seven days, and allows for fibroblast ingrowth into the dermal replacement layer.

73
Q

A 20-year-old man undergoes decompressive craniectomy after a traumatic brain injury. He is now recovered from the injury. The soft tissue overlying the cranial defect is healthy and there is no communication with the paranasal sinuses. Cranioplasty with an alloplastic material is planned. A material with good osteoconductive properties is desired. Which of the following is the most appropriate choice?

A) Hydroxyapatite
B) Polyetheretherketone
C) Polymethylmethacrylate
D) Porous polyethylene
E) Titanium

A

The correct response is Option A.

The ideal material for cranioplasty is biocompatible, radiolucent, heat resistant, inexpensive, and has low risk for infection and extrusion. A healthy and well-perfused soft-tissue envelope is a prerequisite for alloplastic cranioplasty. Moreover, alloplasts in contact with sinuses have a high risk for infection.

Hydroxyapatite (HA) is a calcium phosphate compound with chemical structure similar to that of bone. Of the current alloplastic materials used for cranioplasty, it has the highest capacity for osteoconduction. It is available as a powder that is mixed with liquid to turn it into a malleable form that can be molded to the shape of the defect. Custom made HA ceramic implants can also be produced based on specifications of the defect. The major disadvantage of this material is that it is brittle and may fracture when stressed. It is sometimes used with a titanium mesh underlay to provide more structural stability.

Polymethylmethacrylate (PMMA) is a polymerized ester of acrylic acid. It is the most commonly used alloplastic material for cranioplasty. It is prepared in the operating room via an exothermic reaction; therefore, care has to be taken to protect the dura from thermal injury. Custom implants can also be produced. It can be easily contoured to the shape of the defect and has good mechanical strength. It does not have osteoconductive properties.

Porous polyethylene (PPE) is composed of high-density polyethylene microspheres with an interconnected porous structure. This allows tissue ingrowth in the implant. PPE is radiolucent and has been found to have an excellent safety profile. It does not have osteoconductive properties.

Polyetheretherketone (PEEK) is a semi-crystalline organic thermoplastic polymer. It is lightweight, strong, radiolucent, and heat resistant. Because of these properties, it is becoming a popular material for cranioplasty as a customized implant. It does not have osteoconductive properties.

Titanium is used as an alloy with aluminum and vanadium. It is biocompatible, non-corrosive, radiopaque, and heat resistant. Titanium mesh can be cut to the shape of the defect. It can also be used as a customized implant. The major advantages are its strength, ease of use, and low infection rate. The major disadvantage is its cost. It does not have osteoconductive properties

74
Q

Which of the following processes involves the transformation of recipient mesenchymal cells into osteoprogenitor cells resulting from the stimulation of bone morphogenetic protein?

(A) Endochondral ossification
(B) Membranous ossification
(C) Osteochondrosis
(D) Osteoconduction
(E) Osteoinduction

A

The correct response is Option E.

Osteoinduction describes the process by which tissue types induce cellular differentiation through their actions on each other. Bone morphogenetic protein is stimulated to induce the transformation of perivascular mesenchyme-like cells, known as pericytes, into osteoprogenitor cells.

Endochondral ossification involves the formation of new bone within a hyaline cartilage framework in the epiphysis of the long bones. The process of membranous ossification is primarily responsible for bone formation in the cranial vault and face; this process involves condensation of mesenchymal tissue. Osteochondrosis describes a group of ossification disorders in children. These disorders, which may affect solitary or multiple sites of ossification, are characterized by degeneration of aseptic necrosis of bone followed by reossification. Osteoconduction is the process of tissue ingrowth from the host recipient bed into the grafted material; the bone-producing osteoprogenitor cells play a crucial role in this process.

75
Q

According to Wolff’s law, which of the following factors is critical to the long-term survival of grafted bone?

(A) Presence of membranous bone
(B) Preservation of the periosteum
(C) Preservation of vascularity
(D) Stress

A

The correct response is Option D.

Wolff’s law states that stress is necessary for preservation of the strength and volume of grafted bone. This law has been used to correctly predict the resorption of bone grafted to heterotopic recipient sites and areas lacking the required stress.

Although membranous bone grafts have shown increased long-term survival rates when compared with endochondral grafts, this difference is believed to result from differences in bony architecture as opposed to embryologic origin.

Preservation of an intact periosteum increases graft survival at all stages following transplantation. Delayed revascularization and decreased peripheral bone growth have been demonstrated in bone grafted without periosteum.

Vascularized bone is used for flaps and not for grafts.

76
Q

In a patient undergoing lip enhancement using sheet acellular dermal homograft (Alloderm), which of the following is the correct anatomic placement of the graft?

(A) Subdermal placement along the white roll of the lip
(B) Submucosal placement along the white roll of the lip
(C) Submucosal placement along the wet/dry vermilion border of the lip
(D) Intramuscular placement

A

The correct response is Option C.

Following adequate anesthesia, sheet acellular dermal homograft (Alloderm) should be placed submucosally along the wet/dry vermilion border of the lip. In order to effectively enhance the lips, the surgeon should first create bilateral incisions approximately 0.5 cm from the commissure on both the upper and lower lips; this will allow for tunneling of the Alloderm along this border. After the Alloderm is placed, the lip is stretched, allowing proper sealing of the graft. The ends should then be tapered and placed in a submucosal pocket near the commissure. Suturing is associated with the development of dynamic lip deformities and thus should not be performed.

Because subdermal placement is too superficial, the patient will be predisposed to the development of contour irregularities if the implants are placed at this level. Submucosal placement of the Alloderm along the white roll will result in unnatural lip aesthetics. Alloderm should not be placed within the muscle.

77
Q

Which of the following types of skin graft can be expected to grow proportionately with a young child?

(A) Split-thickness
(B) Full-thickness
(C) Epidermal
(D) Cultured epithelial autograft

A

The correct response is Option B.

Full-thickness skin grafts can be expected to grow proportionately with a young child. In contrast, split-thickness skin grafts will exhibit some growth, although secondary and/or revision grafting is often required. Epidermal grafts and cultured epithelial autografts will not demonstrate proportionate growth with expansion of the surrounding tissues.

78
Q

Which of the following is a characteristic of unilaminar skin substitutes?

(A) Increased bacterial counts within the wound
(B) Inhibition of granulation tissue
(C) Poor fluid absorption
(D) Poor mechanical protection

A

The correct response is Option D.

Although unilaminar skin substitutes have been proven to aid in wound debridement and fluid absorption, as well as to decrease the bacterial count within the wound and stimulate granulation tissue, the mechanical protection provided by this synthetic material is poor. Other skin substitutes such as hydrocolloids, hydrogels, and vapor-permeable membranes have been shown to have similar properties.

79
Q

Bovine collagen is most effective when injected into which of the following anatomic regions?

(A) Epidermis
(B) Dermis
(C) Immediate subdermis
(D) Subcutaneous fat

A

The correct response is Option B.

Bovine collagen provides the most effective aesthetic result when it is injected into the dermal layer. If injected too deeply (ie, into the subdermal or subcutaneous layers), its effects are highly transitory because resorption is often immediate. However, even with appropriate injection, the desired effect of bovine collagen can only be maintained for a maximum of three to four months, and repeat injections are required for a sustained effect. Injection of bovine collagen into the epidermal layer is often associated with contour deformities.

80
Q

Which of the following is most characteristic of hydroxyapatite bone cement?

(A) Exothermic damage to the underlying dura and brain tissue
(B) Gradual loss of contour over time
(C) Osteoinductive growth of new bone
(D) Peripheral ingrowth of bone

A

The correct response is Option D.

Hydroxyapatite bone cement has been shown to have osteoconductive properties, resulting in growth of new bone over several months following its use in reconstruction. This bone cement is a mixture of amorphous and crystalline calcium phosphate compounds and is recommended for use in nonstress skeletal areas. Because the cement sets endothermically at body temperature, patients are not at risk for any endothermic reactions such as those seen with the use of methylmethacrylate cement. It can be molded and injected into various sites and has not been shown to interfere with craniofacial growth in children. In addition, it maintains its original contour over time. Because the x-ray defraction spectrum of hydroxyapatite cement is similar to bone, scatter effect is not seen on CT scan.

81
Q

A deep split-thickness skin graft that has been harvested from the scalp has approximately two thirds of the follicular unit (shaft) within the dermis. The percentage of the follicular unit that can be expected to produce hair growth is closest to

(A) 0%
(B) 30%
(C) 50%
(D) 85%
(E) 100%

A

The correct response is Option B.

If the upper two thirds of the follicular unit (shaft) are transplanted, approximately 30% of the follicles will produce new hair growth. Although the new hairs will be thinner, normal coloring can be expected. The middle third of the hair follicle is partially responsible for hair growth; in contrast, the hair bulb, which in the past was believed to be responsible for hair growth, is actually not required in order to grow new hairs. The bulb is located in the subcutaneous fat beneath the dermis and does contribute somewhat to the growth and health of the hair but is not a necessary factor for growth.

82
Q

Which of the following graft types exhibits the lowest relative volume loss and resorption?

(A) Bone graft
(B) Cartilage graft
(C) Macro-fat graft
(D) Micro-fat graft
(E) Muscle graft

A

The correct response is Option B.

Because of its unique composition, cartilage is a tissue well formulated for grafting due to its minimal volume loss and resorption. The metabolic rate of cartilage resorption is far less than other human tissues, and its glycolic activity and relatively low consumption rate result from a decreased quantity of cells and a relatively isolated cartilage matrix. Sites that are easily accessible for harvest include the auricular cartilage, rib cartilage, and septum.

Rates of resorption following bone grafting typically depend on graft size and clinical variables. Macro-fat grafts are often unreliable and associated with high rates of resorption. Dermal fat grafts are more commonly used because of their lower rates of resorption and fibrosis. Although micro-fat grafts obtained by injection may be more reliable, resorption rates have been reported as 25% to 50% of the volume of the originally injected fat. Muscle alone is not routinely transferred as a graft; vascularized muscle is more often used in flaps.

83
Q

Which of the following characteristics of a full-thickness skin graft has the greatest effect on inhibition of wound contraction?

(A) Epidermal-to-dermal ratio
(B) Percentage of grafted dermis
(C) Presence of muscle at the base of the recipient bed
(D) Skin thickness of the recipient bed
(E) Thickness of the entire graft

A

The correct response is Option B.

Full-thickness skin grafts inhibit wound contraction by accelerating the rate of dissolution of myofibroblasts from the wound. Because of this, it is the percentage of grafted dermis, rather than the absolute thickness of the total graft, that has the greatest effect on inhibition of wound contraction.

The epidermal-to-dermal ratio, skin thickness of the recipient bed, and presence of muscle within the recipient bed play only minor roles in inhibiting wound contraction.

84
Q

Following split-thickness skin grafting, which of the following dressings can be used at the donor site to minimize discomfort, reduce the risk for infection, and decrease healing time?

(A) Bismuth tribromophenate-impregnated gauze (Xeroform)
(B) Heterograft
(C) Hydrocolloid polymer complex (DuoDerm)
(D) Silicone membrane-nylon fabric composite (Biobrane)
(E) o-Tolylazo-_-naphthol- (Scarlet Red-) impregnated gauze

A

The correct response is Option C.

An occlusive dressing consisting of a polyurethane foam and a hydrocolloid polymer complex (DuoDerm), or a semiocclusive dressing consisting of synthetic adhesive moisture vapor permeable films (eg, Op-Site, Tegaderm) will minimize patient discomfort, reduce the risk for infection, and decrease healing time.

Fine mesh gauzes (eg, Scarlet Red, Vaseline, Xeroform) use the semiopen technique of wound healing. Epithelialization and infection rates are favorable, but, when compared with other dressings, pain and discomfort are greater.

Another version of the semiopen wound-healing technique involves the use of a semipermeable silicone membrane and a knitted nylon fabric covalently bonded to porcine collagen (Biobrane). This method is more comfortable for the patient but is associated with a higher rate of infection following skin grafting.

Biologic dressings (eg, cadaveric skin homografts, heterografts, porcine xenografts, amniotic membranes) are frequently associated with marked inflammation of the wound, delayed epithelialization, and prolonged wound healing times.

85
Q

Prior to use, testing for sensitivity to which of the following should be performed?

(A) Acellular dermal homograft (Alloderm)
(B) Botulinum toxin (Botox)
(C) Bovine collagen (Zyderm)
(D) Fat
(E) Polytetrafluoroethylene (Gore-Tex)

A

The correct response is Option C.

Prior to treatment, sensitivity testing to collagen should be performed in an inconspicuous location. Following intradermal injection of a test dose of bovine collagen (Zyderm), the patient should be monitored for four weeks because a hypersensitivity reaction may be delayed. A hypersensitivity reaction is defined as erythema, induration, tenderness, or swelling to any degree, with or without pruritus, that appears more than 24 hours after implantation and/or persists longer than six hours. Symptoms develop in approximately 3% of patients who receive an intradermal test dose. Collagen injections are contraindicated in these patients. Because patients may develop sensitivity to collagen even after successful prior treatment, sensitivity testing should be performed before every treatment.

Acellular dermal homograft (Alloderm) is comprised of processed acellular cadaveric dermis and can be injected without prior skin testing. Because fat is autologous tissue, sensitivity testing is also not required. Polytetrafluoroethylene (Gore-Tex) is a well-tolerated prosthetic material, and sensitivity screening is not indicated prior to its use.

Botulinum toxin should not be injected in persons who have a known hypersensitivity to botulinum toxin or to eggs and/or albumin. The rate of development of antibody-mediated sensitivity or resistance to desired effect with repeat injections is low. A minimum interval of three months between injections is recommended. Currently, there is no indication for testing before use of botulinum toxin.

86
Q

Which of the following is the most common complication of autologous fat grafting?

(A) Contour irregularities
(B) Infection
(C) Migration
(D) Undercorrection

A

The correct response is Option D.

Inadequate correction is the most common complication of autologous fat grafting. This can result from inadequate graft volume initially or graft resorption following the procedure. Graft retention rates vary from 40% to 78% in patients receiving injections in the face; the greatest amount of resorption occurs along the nasolabial lines, and the least resorption occurs in the buccal fat pad. Additional grafting is recommended in those patients who exhibit an undercorrected appearance.

Contour irregularities are caused by misplacement of manipulation grafts. Graft appearance varies according to the quantity of material injected and thickness of the overlying skin.

Infection is commonly associated with violation of the oral mucosa during the infiltration process. Only 1% of patients undergoing autologous fat grafting develop Staphylococcus or Streptococcus infection at either the donor or the injection site.
Graft migration can result from excess pressure at the injection site, infiltration into areas of tension, such as scars or folds, or postoperative manipulation.

Excess correction or overcorrection may also occur, and is difficult to remedy. Because the grafts are infiltrated throughout multiple layers (many of them nonanatomic), it may be impossible to remove the grafts without disturbing the surrounding tissues.

87
Q

Which of the following soft-tissue fillers is derived from hyaluronic acid?

(A) Dermalogen
(B) Fascian
(C) Isolagen
(D) Restylane
(E) Zyderm

A

The correct response is Option D.

Restylane is a cross-linked, stabilized, third-generation hyaluronic acid gel that is a byproduct of processed bacteria. Hyalform gel is also comprised of hyaluronic acid, but is culled instead from an animal-based compound. When used as soft-tissue filler, these substances provide low antigenicity, soft texture, and durability. However, they have not yet been approved for use in the United States.

Dermalogen (dermis) and Fascian (fascia lata) are homologous sources of injectable filler derived from human tissue. Isolagen is an autologous agent derived from human skin cells that consists of cultured fibroblast media with an extracellular matrix. Zyderm is an injectable filling agent derived from bovine collagen.

88
Q

Which of the following soft-tissue fillers is derived from hyaluronic acid?

(A) Dermalogen
(B) Fascian
(C) Isolagen
(D) Restylane
(E) Zyderm

A

The correct response is Option D.

Restylane is a cross-linked, stabilized, third-generation hyaluronic acid gel that is a byproduct of processed bacteria. Hyalform gel is also comprised of hyaluronic acid, but is culled instead from an animal-based compound. When used as soft-tissue filler, these substances provide low antigenicity, soft texture, and durability. However, they have not yet been approved for use in the United States.

Dermalogen (dermis) and Fascian (fascia lata) are homologous sources of injectable filler derived from human tissue. Isolagen is an autologous agent derived from human skin cells that consists of cultured fibroblast media with an extracellular matrix. Zyderm is an injectable filling agent derived from bovine collagen.

89
Q

Which of the following is the most common unfavorable result of lip augmentation with acellular dermal homograft?

(A) Exposure
(B) Hematoma
(C) Infection
(D) Rejection
(E) Resorption

A

The correct response is Option E.

Graft resorption has been reported as the most common complication of lip augmentation with acellular dermal homograft (Alloderm), a product derived from human cadaveric skin for use in soft-tissue augmentation. During harvest of the graft, the dermal and extracellular matrices are left intact, while the immunogenic components are removed. Alloderm can be used for skin grafting of burn wounds, nasal reconstruction, replacement of gingiva, and facial augmentation. Good results have been reported with the use of Alloderm for lip augmentation. Complications other than resorption, such as graft exposure or rejection, hematoma, and infection, have been reported to be minimal.

90
Q

Injection of autologous fat at which of the following sites is associated with increased risk for fat embolism and subsequent blindness and/or central nervous system damage?

(A) Forehead
(B) Glabella
(C) Lateral orbit
(D) Nasolabial fold
(E) Tear trough

A

The correct response is Option B.

Although injection of autologous fat during aesthetic and reconstructive procedures is typically a safe procedure, adverse effects, such as fat embolism and central nervous system damage, have been reported. The surgeon should be particularly cautious when injecting fat into the glabellar region, as the ophthalmic artery, which connects directly to the glabellar vasculature, can be inadvertently divided. Methods to help minimize complications of fat injection include the use of blunt tip large bore cannulas, as well as retrograde injection techniques.

Although the risk for injury exists with injection into the periorbital and nasal regions, it is less than that seen with injection into the glabellar region. Injection into the region of the frontalis muscle (ie, forehead), or into crow’s feet in the region of the lateral orbit, a tear trough, or the nasolabial fold is associated with a lower risk for fat embolism than injection into the glabellar region.

91
Q

What is the lowest bacterial count that will result in infection of cultured epithelial autografts?

(A) 101
(B) 102
(C) 103
(D) 104
(E) 105

A

The correct response is Option B.

Tissue-cultured grafts such as cultured epithelial autografts have a low degree of resistance to infection. They will tolerate bacterial counts to a maximum of 102 to 103 cm3 before developing an infection, as compared with 104 to 105 cm3 for split-thickness skin grafts.

Tissue-cultured skin grafts are expensive and fragile. Well vascularized beds are required for successful graft survival.

92
Q

In a patient undergoing lip enhancement using sheet acellular dermal homograft, which of the following is the correct anatomic placement of the graft?

(A) Subdermal placement along the white roll of the lip
(B) Subdermal placement along the wet/dry vermilion border of the lip
(C) Submucosal placement along the white roll of the lip
(D) Submucosal placement along the wet/dry vermilion border of the lip
(E) Intramuscular placement

A

The correct response is Option D.

Following adequate anesthesia, sheet acellular dermal homograft (Alloderm) should be placed submucosally along the wet/dry vermilion border of the lip. In order to effectively enhance the lips, the surgeon should first place bilateral incisions approximately 0.5 cm from the commissure on both the upper and lower lips; this will allow for tunneling of the Alloderm along this border. After the Alloderm is placed, the lip is stretched, allowing proper sealing of the graft. The ends should then be tapered and placed in a submucosal pocket near the commissure. Suturing is associated with the development of dynamic lip deformities and thus should not be performed.

Because subdermal placement is too superficial, the patient will be predisposed to the development of contour irregularities if the implants are placed at this level. Submucosal placement of the Alloderm along the white roll will result in unnatural lip aesthetics. Alloderm should not be placed within the muscle.